CME Study Part 11(a)

¡Supera tus tareas y exámenes ahora con Quizwiz!

A 20-year-old male presents with complaints of abdominal pain and diarrhea. He says he often has abdominal cramping that is relieved with defecation. The pain is accompanied by frequent loose, mucous stools, and his symptoms tend to get worse with stress. He says he has tried antidiarrheal medications and antispasmodics, but did not get satisfactory results. Your evaluation leads to a diagnosis of diarrhea-predominant irritable bowel syndrome. Which one of the following would be the most appropriate treatment? (check one) A. Fiber supplements B. Neomycin C. Citalopram (Celexa) D. Alosetron (Lotronex) E. Lubiprostone (Amitiza)

Correct A Cochrane review of 15 studies involving 922 patients showed that antidepressants had a beneficial effect on the symptoms of irritable bowel syndrome (IBS). Both SSRIs and tricyclic antidepressants have shown benefit. Another Cochrane review of 12 randomized, controlled trials did not show any benefit from the use of fiber in any type of IBS. Antibiotics have been shown to have some beneficial effects, but neomycin is used only in constipation-predominant IBS. Lubiprostone is a selective C-2 chloride channel activator and can be used for patients with chronic constipation. Alosetron is a 5-hydroxytryptamine 3 antagonist and is FDA approved to treat severe diarrhea-predominant IBS only in women who have not improved with conventional therapy. Alosetron is associated with uncommon but serious adverse events (ischemic colitis, constipation, death) and its use is restricted in the United States. Other potentially beneficial therapies for IBS include peppermint oil, psychological treatments, exercise, and probiotics.

A 72-year-old male is brought to your office by a friend because of increasing confusion, irritability, and difficulty walking. This began shortly after the patient's car broke down in a rural area and he had to walk a mile to get to a phone and call the friend. The temperature outdoors has been near 100°F.On examination the patient has a rectal temperature of 39.5°C (103.1°F), a pulse rate of 110 beats/min, and a blood pressure of 100/60 mm Hg. His shirt is still damp with sweat. Which one of this patient's findings indicates that he has heatstroke rather than heat exhaustion? (check one) A. Confusion B. Sweating C. His temperature D. His heart rate E. His blood pressure

Correct A Heat exhaustion and heatstroke are both on the continuum of heat-related illness. Heatstroke is a much more severe condition than heat exhaustion. Evidence of central nervous system dysfunction is evidence of heatstroke rather than heat exhaustion, even if other symptoms are not severe and point to heat exhaustion. Heatstroke is a medical emergency.

Misleadingly low serum sodium can be caused by? (check one) A. Hyperglycemia B. Diuretic use C. Heart Failure D. Renal Disease

Correct A decrease in serum sodium concentration does not always indicate a decrease in osmolality of body fluids. In cases of hyperglycemia, the main cause of the hyponatremia is the glucose-related increase in osmolality of extracellular fluid, followed by the movement of water from intracellular to extracellular fluid compartments and a subsequent loss of excessive extracellular fluid and electrolytes. The serum sodium concentration is also diminished in patients with hyperlipidemia or hyperproteinuria because of the volume occupied by the lipids or proteins. If the lipids or proteins are removed, the sodium concentration in the remaining plasma is found to be normal. No treatment is needed for these conditions.

A 15-year-old male is seen in the office for ankle pain. While playing basketball he jumped and landed on the lateral edge of his foot. He had immediate pain and did not continue playing, but was able to walk after the injury. On examination his right ankle has tenderness, swelling, and bruising over the anterior talofibular and calcaneofibular ligaments. There is no bony tenderness. Which one of the following would be most appropriate at this point? (check one) A. Taping the ankle for future sports participation B. An elastic compression wrap C. A lace-up ankle support D. A radiograph of the ankle E. A below the knee cast

Correct A lace-up ankle support reduces pain and recovery time after an ankle sprain (SOR B). The Ottawa Rules state that radiography is required only if there is pain in the malleolar or midfoot zone and either bony tenderness over an area of potential fracture (i.e., distal fibula or tibia, lateral or medial malleolus, base of the fifth metatarsal, or navicular bone) or an inability to bear weight immediately after the injury and when evaluated by a physician. This patient did not have those findings, and therefore would not need a radiograph (SOR A). A cast is not necessary for an ankle sprain. An elastic compression wrap alone is not as effective as a lace-up support. Taping of the ankle for future sports participation can reduce the risk of ankle sprains during sports, but would not be appropriate for an acute injury.

A 45-year-old female presents to your office with knee pain. She was playing volleyball yesterday when she collided with another player and was unable to continue playing because of pain in her knee. The knee was swollen this morning. She is able to walk but not without pain, and she also has pain when she attempts to bend her knee. On examination there is medial joint line tenderness and a positive Thessaly test. Which one of the following is the most likely cause of her knee pain? (check one) A. Osteoarthritis B. Anterior cruciate ligament tear C. Collateral ligament tear D. Medial meniscus tear E. Tibial plateau fracture

Correct A medial meniscus tear is the most likely diagnosis in a patient older than 40 who was bearing weight when the injury occurred, was unable to continue the activity, and has a positive Thessaly test. This test is performed by having the patient stand on one leg and flex the knee to 20°, then internally and externally rotate the knee. The presence of swelling immediately after the injury makes an internal derangement of the knee more likely, so osteoarthritis is less probable. This patient is able to bear weight, so a fracture is also not likely. Either a collateral ligament tear or an anterior cruciate ligament tear is possible, but these are not as common in this situation.

[4] A neurologically intact 77-year-old female presents with severe low back pain following a fall 2 days ago that caused her to land on her buttocks. A radiograph of her lower spine shows a compression fracture of L3 with a loss of about 50% of the vertebral body height. Which one of the following is most appropriate at this point? (check one) A. Referral for kyphoplasty B. Referral for vertebroplasty C. Back bracing D. Bed rest E. Calcitonin-salmon (Miacalcin)

Correct A number of measures for managing spinal compression fractures have been evaluated. The evidence for recommending kyphoplasty is weak, and the evidence for recommending against vertebroplasty is strong. The data on bracing is inconclusive, as is the recommendation for bed rest. Calcitonin has been shown to reduce the incidence of recurrent fractures and may be useful in the relief of pain.

A 32-year-old male presents to an urgent care center with a 2-day history of left calf pain and swelling, which started gradually a few hours after he played tennis. He remembers that he "tweaked" his calf on a serve late in the match but was able to continue playing. He has no history of prior medical problems, and no recent surgery or immobilization. On examination his left calf appears slightly erythematous and swollen from the mid-calf to the ankle, with 1+ pitting over the lower leg. There is no venous distention. The left calf is 3 cm greater in circumference than the right calf. He has pain with dorsiflexion, and there is an area of tenderness in the medial calf. Which one of the following is the most appropriate next step in ruling out deep vein thrombosis in this patient? (check one) A. D-dimer B. Ultrasonography C. Venography D. Impedance plethysmography

Correct A number of pretest probability scoring systems are available for assessing venous thromboembolism (VTE), which includes deep vein thrombosis (DVT) and pulmonary embolism. Although the Wells clinical prediction rule is widely used, other tools such as the Hamilton score and the AMUSE (Amsterdam Maastricht Utrecht Study on thromboembolism) score are also available. The Wells rule divides patients suspected of having a DVT into low, intermediate, and high-risk categories, with a 5%, 17%, and 53% prevalence of DVT, respectively. This patient has a Wells score of 0 (+1 for calf circumference increase >3 cm, +1 for pitting edema, -2 for a likely alternative diagnosis of gastrocnemius strain) and is therefore at low risk. A negative D-dimer assay has a high negative predictive value for DVT, so the diagnosis can be ruled out in a patient who has a low pretest probability and a negative D-dimer result. A negative D-dimer assay does not rule out DVT in a patient with a moderate to high pretest probability (SOR C).

When compared with an occiput anterior fetal position, a persistent occiput posterior fetal position is less likely to result in? (check one) A. Spontaneous vaginal delivery B. Assisted vaginal delivery C. Cesarean delivery D. A third or fourth degree perineal laceration E. Excessive maternal blood loss

Correct A persistent occiput posterior position is associated with a higher risk of cesarean delivery and assisted vaginal delivery, and a lower chance of spontaneous vaginal delivery. Assisted vaginal deliveries are associated with a higher rate of third-and fourth-degree perineal lacerations and postpartum hemorrhage.

A 17-year-old football player presents to your office on a Monday with a right knee injury. He injured the knee in Friday night's game when an opposing player fell against the knee from the front while the patient had his right foot planted. He was unable to bear weight after the injury, and noted immediate swelling of the knee. A positive result with which one of the following would indicate an anterior cruciate ligament tear? (check one) A. Ballottement test B. Lachman test C. McMurray test D. Posterior drawer test E. Thessaly test

Correct A positive Lachman test indicates that the anterior cruciate ligament may be torn. The posterior drawer test evaluates posterior cruciate ligament stability. The McMurray and Thessaly assessments test for meniscal tears. The ballottement test is for detecting intra-articular knee effusion.

A 39-year-old female presents with lower abdominal/pelvic pain. On examination, with the patient in a supine position, you palpate the tender area of her lower abdomen. When you have her raise both legs off the table while you palpate the abdomen, her pain intensifies. Which one of the following is the most likely diagnosis? (check one) A. Appendicitis B. A hematoma within the abdominal wall musculature C. Diverticulitis D. Pelvic inflammatory disease E. An ovarian cyst

Correct A reduction of the pain caused by abdominal palpation when the abdominal muscles are tightened is known as Carnett's sign. If the cause of the pain is visceral, the taut abdominal muscles may protect the locus of pain. In contrast, intensification of pain with this maneuver points to a source of pain within the abdominal wall itself.

A 55-year-old male has New York Heart Association class III chronic systolic heart failure due to hypertensive cardiomyopathy. Which one of the following is CONTRAINDICATED in this patient? (check one) A. Carvedilol (Coreg) B. Digoxin C. Ramipril (Altace) D. Spironolactone (Aldactone) E. Verapamil (Calan)

Correct ACE inhibitors and β-blockers improve mortality in heart failure (HF). Digoxin and furosemide improve symptoms and reduce hospitalizations in systolic HF, and furosemide may decrease mortality. Spironolactone, an aldosterone antagonist, reduces all-cause mortality and improves ejection fractions in systolic HF. Verapamil, due to its negative inotropic effect, is associated with worsening heart failure and an increased risk of adverse cardiovascular events. Ref: Jessup M, Abraham WT, Casey DE, et al: 2009 focused update: ACCF/AHA Guidelines for the Diagnosis and Managementof Heart Failure in Adults: A report of the American College of Cardiology Foundation/American Heart Association TaskForce on Practice Guidelines. Circulation 2009;119(14):1977-2016.

Which one of the following is best for preventing acute mountain sickness? (check one) A. Acetazolamide (Diamox Sequels) started the day before arriving at altitude B. Prednisone started the day before arriving at altitude C. Moderate alcohol consumption on the first day at altitude D. Ascending quickly, then resting to acclimatize before beginning planned activities

Correct Acute mountain sickness is common in people traveling to altitudes higher than 8200 ft. Symptoms include headache and at least one of the following: nausea or vomiting, anorexia, dizziness or lightheadedness, fatigue or weakness, and difficulty sleeping. Slow ascent is the most effective way to prevent acute mountain sickness. Acetazolamide or dexamethasone can be used for both prevention and treatment. Ataxia and altered mental status are signs of cerebral edema and occur with end-stage acute mountain sickness. This can progress to coma and death and requires prompt treatment and descent. High-altitude pulmonary edema can occur without acute mountain sickness. Alcohol consumption on the first day at altitude can exacerbate acute mountain sickness.

A 44-year-old male in the intensive-care unit develops acute respiratory distress syndrome (ARDS). Which one of the following has been shown to improve outcomes in this situation? (check one) A. Surfactant B. Lower positive end-expiratory pressure (PEEP) settings C. Lower tidal volumes D. Aggressive fluid therapy E. Pulmonary artery catheters

Correct Acute respiratory distress syndrome (ARDS) may be caused by pulmonary sepsis or sepsis from another source, or it may be due to acute pulmonary injury, including inhalation of smoke or other toxins. Inflammatory mediators are released in response to the pulmonary infection or injury. The syndrome has an acute onset and is manifested by rapidly developing profound hypoxia with bilateral pulmonary infiltrates. The mortality rate in patients with ARDS may be as high as 55%. Early recognition and prompt treatment with intubation and mechanical ventilation is necessary to improve chances for survival. Patients with ARDS should be started at lower tidal volumes (6 mL/kg) instead of the traditional volumes (10-15 mL/kg) (SOR A). These patients also often require higher positive end-expiratory pressure settings (SOR B). Fluid management should be conservative to allow for optimal cardiorespiratory and renal function and to avoid fluid overload. However, the routine use of central venous or pulmonary artery pressure catheters is not recommended due to the potential complications associated with their use (SOR A). While surfactant is commonly used in children with ARDS, it does not improve mortality in adults (SOR A).

A 44-year-old female is brought to your office by her mother. The patient was in a severe car accident 2 weeks ago. Her husband was killed instantly and she was extracted by emergency responders almost an hour later. She received a full examination at a local emergency department and was discharged home with only minor contusions and abrasions and no evidence of a closed head injury. The patient has been panicked and unable to sleep. She has recurrent flashbacks of the event and dreams repeatedly about her husband's death. She says that sometimes, even while awake, she can almost sense her husband's lifeless body near her. She has refused to get into a car since the accident, which is the reason she has not sought care sooner. She has not been able to focus on daily tasks but has been able to eat and drink adequate amounts. Which one of the following diagnoses best describes her condition? (check one) A. Acute stress disorder B. Major depressive disorder C. Obsessive-compulsive disorder D. Panic disorder E. Generalized anxiety disorder

Correct Acute stress disorder (ASD) lies on a spectrum of trauma-related disorders between adjustment disorder and posttraumatic stress disorder (PTSD). ASD is differentiated from PTSD primarily by duration, with PTSD requiring the presence of similar symptoms (intrusion, negative mood, dissociation, avoidance, and arousal) for longer than 1 month. Conversely, adjustment disorder is a less severe condition than ASD that involves either a less traumatic or threatening inciting event and/or less severe symptoms that do not meetDSM-5 criteria for acute stress disorder.

A 35-year-old white female complains of severe pain in her right shoulder. She notes that at night the pain intensifies if she rolls onto her right side. She has marked pain with no weakness on abduction of the shoulder; range of motion of the shoulder is normal. The most likely cause of her pain is? (check one) A. Bicipital tendinitis B. Complete rotator cuff tear C. Osteoarthritis of the shoulder D. Subacromial bursitis E. Frozen shoulder

Correct Acute subacromial bursitis is common and is often associated with calcific deposits in the supraspinatus tendon, pain on abduction, and local tenderness. Bicipital tendinitis results in tenderness on palpation of the tendon of the long head of the biceps. A rotator cuff tear usually results from an injury, and affects range of motion. Osteoarthritis seldom causes acute, severe pain. A frozen shoulder may result from subacromial bursitis and presents with limitation of shoulder motion.

A 30-year-old female sees you because of increasing fatigue. She has no chronic medical problems and reports no recent acute illnesses. She recalls being told that she was mildly anemic after the birth of her daughter 3 years ago. The anemia resolved after 3 months of oral iron supplementation. The patient's menstrual periods are regular and last approximately 6 days, with heavy bleeding for the first 3 days then moderate to mild flow for approximately 3 days. She denies epistaxis, black stools, or other signs of bleeding. On examination her temperature is 36.7°C (98.1°F), pulse rate 93 beats/min, respiratory rate 16/min, and blood pressure 116/58 mm Hg. The remainder of her physical examination is unremarkable. A CBC is notable for a hemoglobin level of 10.9 g/dL (N 12.0-16.0) and a mean corpuscular volume of 70 !m3 (N 78-102). Which one of the following serum levels would be most appropriate for further evaluating her microcytic anemia at this point? (check one) A. Ferritin B. Folate C. Erythropoietin D. Hemoglobin A1c E. TSH

Correct After confirmation of anemia and microcytosis on a CBC, a serum ferritin level is recommended (SOR C). If the ferritin level is consistent with iron deficiency anemia, identifying the underlying cause of the anemia is the priority. A common cause of iron deficiency anemia in premenopausal adult women is menstrual blood loss. If the serum ferritin level is not consistent with iron deficiency anemia, the next stage of the evaluation should include a serum iron level, total iron-binding capacity (TIBC), and transferrin saturation (SOR C). Iron deficiency anemia is still probable if the serum iron level and transferrin saturation are decreased and TIBC is increased. It is more likely anemia of chronic disease if the serum iron level is decreased and the TIBC and transferrin saturation are decreased or normal. Other laboratory tests that may help in differentiating the cause of microcytosis include hemoglobin electrophoresis, a reticulocyte count, and peripheral blood smears.

A 64-year-old male with a previous history of hypertension and atrial fibrillation presents with an acute onset of ataxia, headache, mild confusion, and restlessness. His only current medications are lisinopril (Prinivil, Zestril) and warfarin (Coumadin). On examination his blood pressure is 160/100 mm Hg, pulse rate 86 beats/min, respirations 12/min, and temperature 36.7°C (98.1°F). A CBC, serum electrolyte levels, and cardiac enzyme levels are normal. His INR is 1.1. Noncontrast CT shows a cerebellar hemorrhage with a hematoma volume of 50 mL. Which one of the following should be performed urgently? (check one) A. Neurosurgical consultation for posterior cerebellar hematoma decompression B. A reduction in blood pressure to 140/90 mm Hg C. Administration of vitamin K, 10 mg intravenously D. Administration of mannitol (Osmitrol), 0.5-1.0 mg/kg intravenously E. Induction of hypothermia to achieve a body temperature of 34.4°C (94.0°F)

Correct Aggressive neurosurgical intervention is not indicated to evacuate clots in patients with intracerebral hemorrhage except in those with a cerebellar hemorrhage, which is always an indication for neurosurgical consultation. Guidelines have been developed by the American Heart Association for lowering blood pressure in patients with a systolic blood pressure >180 mm Hg, or a mean arterial pressure >130 mm Hg. The use of various forms of osmotherapy, including mannitol, to prevent the development of cerebral edema has not been shown to improve outcomes. The data regarding hypothermia induction is unclear. Patients with an INR >1.5 should receive therapy to replace vitamin K-dependent factors and have their warfarin withheld.

A 63-year-old male with a history of alcoholism and compensated hepatic cirrhosis asks if there are pain medications he can use to treat his chronic low back pain and knee and hand osteoarthritis. He also has occasional headaches. He has not used alcohol for several years. Which one of the following medications is CONTRAINDICATED in this patient? (check one) A. Acetaminophen B. Gabapentin (Neurontin) C. Naproxen D. Pregabalin (Lyrica)

Correct Although patients with chronic mild liver disease may take NSAIDs, they should be avoided in all patients with cirrhosis, due to the risk of precipitating hepatorenal syndrome. Pregabalin and gabapentin are not metabolized by the liver and can be quite helpful. Acetaminophen, while toxic in high doses, can be used safely in dosages of 2-3 g/day. Tramadol is also safe in patients with cirrhosis.

Which one of the following is a preferred first-line agent for managing hypertension in patients with stable coronary artery disease? (check one) A. A thiazide diuretic B. An angiotensin receptor blocker C. A β-blocker D. A long-acting calcium channel blocker E. A long-acting nitrat

Correct American Heart Association guidelines recommend treating hypertension in patients with stable heart failure with ACE inhibitors and/or β-blockers. Other agents, such as thiazide diuretics or calcium channel blockers, can be added if needed to achieve blood pressure goals (SOR B). β-Blockers with intrinsic sympathomimetic activity should be avoided, as they increase myocardial oxygen demand. While thiazide diuretics are often a first choice for uncomplicated hypertension, this is not the case for patients with coronary artery disease. Long-acting calcium channel blockers may be used in patients who do not tolerate β-blockers, but short-acting calcium channel blockers should be avoided because they increase mortality. ACE inhibitors are recommended as antihypertensive agents in patients already on β-blocker therapy (especially following myocardial infarction), in diabetics, and in patients with left ventricular dysfunction. Although angiotensin receptor blockers have indications similar to those of ACE inhibitors, the American Heart Association recommends using them only in patients who do not tolerate ACE inhibitors. Long-acting nitrates are used for their anti-anginal properties and have no role in the management of hypertension.

Which one of the following is most likely to cause hypoglycemia in elderly patients? (check one) A. Metformin (Glucophage) B. Pioglitazone (Actos) C. Glipizide (Glucotrol) D. Sitagliptin (Januvia) E. Glyburide (DiaBeta)

Correct Among the oral antiglycemic drugs, the sulfonylurea agents are the most likely to cause hypoglycemia, and glyburide is more likely to cause hypoglycemia than glipizide. Glyburide should rarely be used in the elderly.

Which one of the following is considered first-line therapy for mild to moderate Alzheimer's disease? (check one) A. Donepezil (Aricept) B. Memantine (Namenda) C. Selegiline (Eldepryl) D. Risperidone (Risperdal) E. Ginkgo biloba

Correct Anticholinesterase inhibitors such as donepezil are considered first-line therapy for patients with mild to moderate Alzheimer's disease (SOR A). Memantine is an NMDA receptor antagonist and is often used in combination with anticholinesterase inhibitors for moderate to severe Alzheimer's disease, but it has not been shown to be effective as a single agent for patients with mild to moderate disease. There is not enough evidence to support the use of selegiline, a monoamine oxidase type B inhibitor, in the treatment of Alzheimer's disease. Risperidone and other antipsychotic medications are not approved by the Food and Drug Administration for treatment of Alzheimer's disease, but can sometimes be helpful in controlling associated behavioral symptoms. Studies of ginkgo biloba extract have not shown a consistent, clinically significant benefit in persons with Alzheimer's disease.

Prophylactic cholecystectomy for asymptomatic gallstones is indicated for patients with which one of the following? (check one) A. Sickle cell disease B. A renal transplant C. Diabetes mellitus D. Cirrhosis

Correct Asymptomatic gallstones are not usually an indication for prophylactic cholecystectomy, as most patients remain asymptomatic throughout their lives, and only 1%-4% develop symptoms or complications from gallstones each year. Only 10% of patients found to have asymptomatic gallstones develop symptoms within the first 5 years after diagnosis, and only 20% within 20 years. In the past, cholecystectomy was recommended for diabetic patients with asymptomatic gallstones, based on the assumption that autonomic neuropathy masked the pain and signs associated with acute cholecystitis, and that patients would therefore develop advanced disease and more complications. More recent evidence has shown that these patients have a lower risk of major complications than previously thought.Prophylactic cholecystectomy is not recommended in renal transplant patients with asymptomatic gallstones. One study found that 87% of these patients remained asymptomatic after 4 years, with only 7% developing acute cholecystitis and requiring subsequent uncomplicated laparoscopic cholecystectomy. Other studies have shown that the presence of gallstone disease does not negatively affect graft survival. Patients with hemoglobinopathies are at a significantly increased risk for developing pigmented stones. Gallstones have been reported in up to 70% of sickle cell patients, up to 85% of hereditary spherocytosis patients, and up to 24% of thalassemia patients. In sickle cell patients, complications from asymptomatic gallstones have been reported to be as high as 50% within 3-5 years of diagnosis. This has been attributed largely to the diagnostic challenge associated with symptomatic cholelithiasis versus abdominal sickling crisis. In the past these patients were managed expectantly because of the significant morbidity and mortality associated with open operations. The operative risk for these patients (especially sickle cell patients) has been lowered by laparoscopic cholecystectomy, along with improved understanding of preoperative hydration and transfusion, improved anesthetic technique, and better postoperative care. Prophylactic laparoscopic cholecystectomy in these patients prevents future diagnostic confusion, as well as the mortality and morbidity risk associated with emergency surgery. Furthermore, cholecystectomy can and should be performed at the time of splenectomy, whether open or laparoscopic. Studies have shown no significant differences in progression to symptoms from silent gallstones in cirrhotic patients compared with noncirrhotic patients. Expectant management is therefore recommended in patients with cirrhos

A 56-year-old African-American male with long-standing hypertension and a 30-pack-year smoking history has a 2-day history of dyspnea on exertion. A physical examination is unremarkable except for rare crackles at the bases of the lungs. Which one of the following serologic tests would be most helpful for detecting left ventricular dysfunction? (check one) A. B-type natriuretic peptide B. Troponin T C. C-reactive protein (CRP D. D-dime E. Cardiac interleukin-2

Correct B-type natriuretic peptide (BNP) is a 32-amino acid polypeptide secreted from the cardiac ventricles in response to ventricular volume expansion and pressure overload. The major source of BNP is the cardiac ventricles, and because of the minimal presence of BNP in storage granules, its release is directly proportional to ventricular dysfunction. A BNP test is simple and time efficient, and reliably predicts the presence or absence of left ventricular dysfunction on an echocardiogram.

Which one of the following activities is most likely to be impaired in early dementia? (check one) A. Dressing B. Eating C. Toileting D. Grooming E. Cooking

Correct Basic activities of daily living, such as dressing, eating, toileting, and grooming, are generally intact in early dementia. In contrast, instrumental activities of daily living, such as managing money and medications, shopping, cooking, housekeeping, and transportation, which often require calculation or planning, are frequently impaired in early dementia.

A 57-year-old male comes to the emergency department after several episodes of vomiting preceded by moderately severe epigastric pain. He says the vomitus looked like coffee grounds. He tells you he has had "heartburn" in the past that was sometimes severe, and occasionally associated with vomiting, but these episodes were almost always relieved by oral antacids. This problem was exacerbated recently after he began taking ketorolac for moderate arthritic pain in his knees and hands. His past medical history and a review of systems reveal no major comorbid disorders.The patient's blood pressure is 125/82 mm Hg and his heart rate is 95 beats/min with no signs of shock. His hemoglobin level is 9.5 g/dL (N 13.0-18.0). He is admitted to the hospital and placed on a proton pump inhibitor (PPI) infusion. Upper gastrointestinal endoscopy performed within 3 hours of admission shows no blood in the upper gastrointestinal tract, but reveals a Mallory-Weiss tear and a stomach ulcer containing a dark spot in an otherwise clear base. Management at this time should include which one of the following? (check one) A. Transfusion with whole blood B. Repeat endoscopy within 24 hours C. Arteriography D. Continued in-hospital observation for at least 72 hours E. Discharge from the hospital on oral PPI therapy

Correct Blood transfusions should be administered to patients with upper gastrointestinal bleeding who have a hemoglobin level ≤7.0 g/dL (SOR C). According to the Rockall risk scoring system, this patient's mortality risk from gastrointestinal bleeding is low, based on the following: age <60, systolic blood pressure ≥100 mm Hg, heart rate <100 beats/min, no shock, and no major comorbidities. The Mallory-Weiss tear adds no points to his total score, and the only stigmata of recent hemorrhage is a dark spot in an otherwise clean ulcer base, which also adds no points. His only scored finding is the presence of the ulcer, which adds a single point to his score. Patients with low-risk peptic ulcer bleeding based on clinical and endoscopic criteria can be discharged from the hospital on the same day as endoscopy (SOR C). Routine second-look endoscopy is not recommended in patients with upper gastrointestinal bleeding who are not considered to be at high risk for rebleeding (SOR C). Arteriography with embolization is indicated only in patients with persistent bleeding.

A 42-year-old male sees you for help to quit smoking. His sister had excellent results with bupropion (Zyban) and he asks if he could try using it. When you review his medical history, which one of the following would be a contraindication to bupropion? (check one) A. Diabetes mellitus B. Gout C. Hypertension D. Hyperthyroidism E. A seizure disorder

Correct Bupropion can lower the seizure threshold and should not be used in patients who have a history of a seizure disorder or who drink heavily. A history of the other medical conditions listed does not contraindicate the use of bupropion.

An 11-year-old female who plays in a local youth soccer league presents with right heel pain that has persisted for several months. She does not recall a specific injury that could have caused the pain. On examination the skin is intact and she is tender over the right posterior heel. Examination of the ankle and forefoot is unremarkable and a neurovascular examination is normal. You decide that rest and physical therapy would be the best initial management. Which one of the following is CONTRAINDICATED for this patient? (check one) A. Ice packs B. Moist heat C. Whirlpool therapy D. An exercise prescription E. Therapeutic ultrasound

Correct Calcaneal apophysitis, also called Sever's disease, is a common cause of heel pain in young athletes, especially those who participate in basketball, soccer, track, and other sports that involve running. Typically the heel apophysis closes by age 15. Treatment options include activity modification, the use of ice packs and/or moist heat, stretching, analgesics, and orthotic devices. The use of therapeutic ultrasound on the active bone growth plates in children is contraindicated.

A 30-year-old female at 36 weeks gestation has a positive culture for group B Streptococcus. Her past medical history is significant for the development of a nonurticarial rash in response to penicillin. Which one of the following is most appropriate for intrapartum antibiotic prophylaxis in this patient? (check one) A. Azithromycin (Zithromax) B. Clindamycin (Cleocin) C. Vancomycin (Vancocin) D. Ampicillin E. Cefazolin

Correct Cefazolin is appropriate for intrapartum prophylaxis against group B Streptococcus (GBS) in penicillin-allergic patients who do not have a history of anaphylaxis, urticaria, angioedema, or respiratory distress. Depending on the antibiotic sensitivity of the GBS organism, either vancomycin or clindamycin is recommended for patients at higher risk for anaphylaxis.

A 24-year-od nulligravida comes to your office for contraception counseling. She has a seizure disorder that is well controlled on carbamazepine (Tegretol). She is a nonsmoker and has no other medical problems or complaints. She is currently in a relationship and does not want to get pregnant in the next several years. Which one of the following contraceptive options would be the most appropriate? (check one) A. Progestin-only pills B. Combined oral contraceptives C. The etonogestrel/ethinyl estradiol vaginal ring (NuvaRing D. The norelgestromin/ethinyl estradiol contraceptive patch (Ortho Evra) E. A levonorgestrel intrauterine device (Mirena)

Correct Certain antiepileptic drugs induce hepatic metabolism of estrogen and progestin (carbamazepine, oxcarbazepine, phenobarbital, phenytoin, and topiramate). This can potentially lead to failure of any contraceptive that contains estrogen and progestin. Progestin-only pills are most effective in women who are exclusively breastfeeding. They are not as effective in pregnancy prevention in other circumstances. Another effective option for women taking antiepileptic medications would be an intrauterine device. The levonorgestrel (progestin only) IUD and copper IUD are acceptable choices even for a nulligravida. The single-rod implantable progestin system also would be an acceptable choice for this patient.

Which one of the following occurs with delirium tremens but is not usually seen with less severe forms of alcohol withdrawal? (check one) A. Fever B. Hypertension C. Tachycardia D. Seizure E. Visual hallucinations

Correct Chronic excessive alcohol intake produces functional changes in neurotransmitter activity that can lead to a net increase in excitatory neuroreceptor activity when the person stops drinking. Withdrawal can be divided into four levels of severity: minor, major, seizures, and delirium tremens. Minor alcohol withdrawal is characterized by tremor, anxiety, nausea, vomiting, and/or insomnia 6-24 hours after the patient's last drink. Major withdrawal occurs 10-72 hours after the last drink and can include the signs and symptoms of minor withdrawal, as well as visual and auditory hallucinations, diaphoresis, tachycardia, and elevated blood pressure. Alcoholic seizure generally occurs within 2 days of the last drink and may be the only sign of withdrawal, although approximately one-third of these patients will progress to delirium tremens. The onset of delirium tremens can occur anytime within 3-10 days following the last drink. The defining clinical finding is delirium, but the findings seen in milder forms of alcohol withdrawal can also be present, and may be more severe. Fever is most often seen with delirium tremens and is less common with less severe forms of alcohol withdrawal.

A 57-year-old female is hospitalized for hypotension. She has stage IV breast cancer with extensive visceral and skeletal metastases. For the past 2 weeks she has had fatigue, nausea, and anorexia. She also reports a 3-lb weight loss during this time. She decided to stop chemotherapy 1 month ago. The patient appears pale with a pulse rate of 78 beats/min and a blood pressure of 82/54 mm Hg. Her physical examination is unremarkable except for lower thoracic spine tenderness on percussion. Laboratory studies reveal a serum sodium level of 132 mEq/L, a potassium level of 5.2 mEq/L, and a hemoglobin level of 10.5 g/L. Chest radiographs reveal scattered pulmonary metastatic lesions. The patient is started on intravenous fluid resuscitation with normal saline. On day 2 her blood pressure continues to remain low despite aggressive fluid replacement. Which one of the following should be administered next to manage her hypotension? (check one) A. Broad-spectrum antibiotics B. Dobutamine C. Dopamine D. Hydrocortisone E. Packed RBCs

Correct Common features of acute adrenal insufficiency include fatigue and lack of energy, weight loss, hypotension, loss of appetite, nausea, and vomiting. Other features such as dry skin, hyperpigmentation, and abdominal pain are seen to varying degrees. Common laboratory findings include electrolyte disturbances, hyponatremia, hyperkalemia, hypercalcemia, azotemia, anemia, and eosinophilia. Patients can also have unexplained hypoglycemia. Patients with advanced-stage cancer (especially of the lung or breast) may develop acute adrenal insufficiency from metastatic infiltration of the adrenal glands. Intravenous hydrocortisone is the treatment of choice in the management of adrenal crisis. For managing hypotension, dopamine is recommended for patients with sepsis, dobutamine for those in cardiogenic shock, and packed RBCs for those with hemorrhagic shock. Broad-spectrum antibiotics are part of the therapy for sepsis, but are not first-line agents for hypotension (SOR B).

A 15-year-old male has a 1-week history of a nonproductive cough, a low-grade fever, a sore throat, and hoarseness. His respiratory rate is 22/min but unlabored, his temperature is 38.1°C (100.6°F), and his O2 saturation is 94% on room air. A chest radiograph reveals bilateral interstitial infiltrates. Which one of the following treatments would be most appropriate for this patient? (check one) A. Ceftriaxone (Rocephin) B. Amoxicillin C. Cefdinir D. Linezolid (Zyvox) E. Azithromycin (Zithromax)

Correct Community-acquired pneumonia in children over the age of 5 is most commonly due to Mycoplasma pneumoniae, Chlamydophila pneumoniae, and Streptococcus pneumoniae. Less common bacterial infections include Haemophilus influenzae, Staphylococcus aureus, and group A Streptococcus. Initial treatment with antibiotics is empiric, as the pathogen is usually unknown at the time of diagnosis. The choice in children is based on age, severity of illness, and local patterns of resistance. Children age 5-16 years who can be treated as outpatients are usually treated with oral azithromycin. For patients requiring inpatient management, intravenous cefuroxime plus either intravenous erythromycin or azithromycin is recommended

Routine vaccination against which one of the following organisms has significantly reduced the risk of bacterial meningitis among young children? (check one) A. Borrelia burgdorferi B. Escherichia coli C. Haemophilus influenzae D. Listeria monocytogenes E. Mycoplasma pneumoniae

Correct Conjugate vaccines against Haemophilus influenzae type b and Streptococcus pneumoniae have been highly effective in reducing the incidence of bacterial meningitis in young children and are now routinely recommended for infants and older patients who fall into appropriate risk groups (SOR B). Escherichia coli and Listeria monocytogenes also cause meningitis in young children, but there is not currently a routine vaccine for these pathogens. Likewise, Borrelia burgdorferi and Mycoplasma pneumoniae can cause aseptic meningitis, but there is no routine vaccine.

An 82-year-old female with terminal breast cancer has been admitted to hospice care. She is having severe pain that you will manage with opioids. Which one of the following would be appropriate to recommend for preventing constipation? (check one) A. Fiber supplements B. Docusate (Colace C. Metoclopramide (Reglan) D. Polyethylene glycol (MiraLax) E. No preventive measures, and treatment only if constipation develops

Correct Constipation is a very common side effect of opioids that does not resolve with time, unlike many other adverse effects. Constipation is easier to prevent than to treat, so it is important to start an appropriate bowel regimen with the initiation of opioid therapy. Fiber supplements and detergents (such as docusate) are inadequate for the prevention of opioid-induced constipation. Metoclopramide is used for nausea and increases gastric motility, but is not indicated in the treatment of constipation. Polyethylene glycol, lactulose, magnesium hydroxide, and senna with docusate are all appropriate in this situation.

A 64-year-old male comes to your office for evaluation of a persistent rash affecting his groin. It is itchy but not painful and does not affect his daily activities. He has tried over-the-counter antifungal creams without relief. On examination you find well-demarcated, dark red patches in the inguinal region bilaterally. When examined with a Wood's light the area fluoresces coral-red. The most effective treatment for this condition is topical? (check one) A. Ketoconazole (Nizoral) B. Erythromycin C. Hydrocortisone D. Mupirocin (Bactroban) E. Terbinafine (Lamisil)

Correct Coral-red fluorescence on Wood's light examination is typical of infection with Corynebacterium minutissimum, a condition known as erythrasma. This organism commonly complicates intertrigo, often in the groin or interdigital spaces. Erythromycin is the most effective treatment for this bacterial infection.

A 72-year-old white female is admitted to the hospital with her first episode of acute heart failure. She has a history of hypertension treated with a thiazide diuretic. An echocardiogram reveals no evidence of valvular disease and no segmental wall motion abnormalities. Left ventricular hypertrophy is noted, and her ejection fraction is 55%. Her pulse rate is 72beats/min. The most likely cause of her heart failure is (check one) A. systolic dysfunction B. diastolic dysfunction C. hypertrophic cardiomyopathy D. high out-put failure

Correct Diastolic dysfunction is now recognized as an important cause of heart failure. It is due to left ventricular hypertrophy as a response to chronic systolic hypertension. The ventricle becomes stiff and unable to relaxor fill adequately, thus limiting its forward output. The typical patient is an elderly person who has systolic hypertension, left ventricular hypertrophy, and a normal ejection fraction (50%-55%).

A 12-year-old female with asthma sees you for a follow-up visit. The girl's mother states that she is currently coughing several days per week and uses her albuterol (Proventil, Ventolin) inhaler 3-4 times weekly. She has awakened with a cough during the night 3 times in the last month. The patient thinks her asthma only mildly affects her day-to-day activity. In-office spirometry reveals that her FEV1 is 83% of predicted, with a normal FEV1/FVC ratio. Which one of the following asthma classifications best fits this patient's presentation? (check one) A. Intermittent B. Mild persistent C. Moderate persistent D. Severe persistent E. Status asthmaticus

Correct Education of asthmatic patients is critically important in their follow-up care. This includes informing patients about the severity of their asthma in addition to instruction about appropriate treatment modalities. The National Heart, Lung, and Blood Institute's National Asthma Education and Prevention Program uses the following definitions for asthma severity: Intermittent: Symptoms less than or equal to twice weekly, nighttime awakenings ≤2 times/month, short-acting β-agonist usage ≤2 days/week, no interference with daily activities, and normal FEV1 and FEV1/FVC ratio at baseline Mild Persistent: Symptoms >2 days/week but not daily, nighttime awakenings 3-4 times/month, short-acting β-agonist usage >2 days/week but not more than once daily, minor limitation to daily activities, FEV1 ≥80% predicted, and normal FEV1/FVC ratio Moderate Persistent: Daily symptoms, nighttime awakenings greater than once weekly but not nightly, daily use of a short-acting β-agonist, some limitation to daily activity, FEV1 >60% but <80% of predicted, and FEV1/FVC ratio reduced by 5% Severe Persistent: Symptoms throughout the day, nighttime awakenings nightly, short-acting β-agonist usage several times daily, extremely limited daily activities, FEV1 <60% of predicted, and FEV1/FVC ratio reduced by >5% Status asthmaticus is a medical emergency and requires emergent treatment in a hospital setting.

You are counseling a 45-year-old obese male regarding weight loss. The patient has elevated triglycerides, low HDL-cholesterol, and stage 1 hypertension. He does not currently take medications and would like to avoid taking medications in the future. The patient has heard good things about low-carbohydrate diets and asks your opinion. A low-carbohydrate diet in a patient such as this is most likely to result in? (check one) A. Increased LDL-cholesterol B. Increased triglycerides C. Increased blood pressure D. Development of metabolic syndrome E. Better short-term weight loss than with a low-fat diet

Correct Emerging data on low-carbohydrate diets is mostly encouraging, in that these diets do not seem to cause the expected increases in blood pressure, LDL-cholesterol levels, or triglyceride levels that the medical community had first assumed. Although low-carbohydrate diets have been shown to result in clinically meaningful weight loss, reduced-calorie diets appear to result in similar weight loss regardless of which macronutrients they emphasize. This patient has symptoms of metabolic syndrome and has a higher risk of glucose intolerance or diabetes mellitus. Low-carbohydrate diets have been shown to reduce insulin resistance at least as well as, if not better than, traditional diet plans.

A mother brings her 10-year-old son to your office because he has recently experienced a flare-up of atopic dermatitis, including increased pruritus. Physical findings include increased erythema of the involved skin on the flexural surfaces of his arms and legs, with weeping eruptions located within areas of lichenification. Which one of the following topical treatments for managing this episode is supported by the best available evidence? (check one) A. Emollients B. Pimecrolimus (Elidel) C. Mupirocin (Bactroban) D. Corticosteroids E. Antihistamines

Correct Emollients are a mainstay of chronic therapy for atopic dermatitis (SOR C), but topical corticosteroids are the first-line treatment for flare-ups (SOR A). Calcineurin inhibitors such as pimecrolimus are a second-line treatment for moderate to severe atopic dermatitis (SOR A). Antibiotics are not useful in reducing flare-ups of atopic dermatitis unless there is clear evidence of a secondary infection (SOR A). Neither topical nor oral antihistamines are recommended for routine treatment of atopic dermatitis because they are not effective in treating the associated pruritus.

You are covering the inpatient service and following up on a 67-year-old female admitted 3 days ago for severe pancreatitis. CT on admission showed edema and mild inflammation. Currently the patient is receiving intravenous fluids, daily laboratory evaluations, and pain medications. She is NPO and afebrile, with a blood pressure of 130/78 mm Hg and a pulse rate of 88beats/min. Which one of the following therapies should be initiated to lower complication rates and shorten the patient's hospital stay? (check one) A. Enteral nutrition B. Parenteral nutrition C. Surgical debridement D. Prophylactic antibiotics

Correct Enteral nutrition is preferred over parenteral nutrition for patients with severe pancreatitis who have been on prolonged bowel rest, and it is associated with lower complication rates and shorter hospitalizations(SOR A). Prophylactic antibiotics should only be used when there is significant necrosis (SOR C).Similarly, surgical debridement is indicated only if there is infected necrosis or persistent fluid collections(SOR C).

The etiologic agent that causes erysipelas is? (check one) A. Staphylococcus aureus B. Haemophilus influenzae C. Streptococcus pyogenes D. Pseudomonas aeruginosa E. Rubivirus

Correct Erysipelas is caused primarily by group A Streptococcus, with a rare case caused by group C or G. Most cases of erysipelas involve the face, but the lesions can occur anywhere on the body. Penicillin is an effective treatment.

A 50-year-old male with difficult-to-control hypertension seeks your advice regarding progressive breast enlargement. Your examination reveals bilateral firm, glandular tissue in a concentric mass around the nipple-areola complex. You diagnose gynecomastia. Which one of the following antihypertensive medications is most likely to cause this problem? (check one) A. Doxazosin (Cardura) B. Hydrochlorothiazide C. Lisinopril (Prinivil, Zestril) D. Losartan (Cozaar) E. Spironolactone (Aldactone)

Correct Except for persistent pubertal gynecomastia, medication use and substance use are the most common causes of nonphysiologic gynecomastia. Common medication-related causes include the use of antipsychotic agents, antiretroviral drugs, or prostate cancer therapies. Spironolactone also has a high propensity to cause gynecomastia; other mineralocorticoid receptor antagonists, such as eplerenone, have not been associated with similar effects. Discontinuing the contributing agent often results in regression of breast tissue within 3 months.

A 58-year-old female with diabetes mellitus complains of 2 years of right shoulder pain, which is worse with activity. There has been no trauma. She tells you one of her friends had a similar problem and was treated successfully with "some sort of shock wave treatments." Which one of the following diagnoses is most likely to be successfully treated with extracorporeal shock wave therapy? (check one) A. Calcific tendinitis B. Gout C. Partial rotator cuff tear D. Frozen shoulder E. Hooked acromion

Correct Extracorporeal shock wave therapy is effective for calcific tendinitis of the rotator cuff. Side effects include bruising and pain. Needling and irrigation, physical therapy, and cortisone injections are sometimes used in patients with acute symptoms. Endoscopic and open surgical treatments are alternatives to extracorporeal shock wave therapy in refractory cases. Extracorporeal shock wave therapy does not have an established role in gout, rotator cuff tear, frozen shoulder, or hooked acromion.

A 49-year-old uninsured female with diabetes mellitus presents with painful burning of her feet, particularly at night. She has tried ibuprofen and acetaminophen without relief. Her last hemoglobin A1c was 7.1%. Her medications include metformin (Glucophage), glipizide (Glucotrol), lisinopril (Prinivil, Zestril), and lovastatin (Mevacor). Which one of the following would be the best choice to treat her foot pain? (check one) A. Amitriptyline B. Topiramate (Topamax) C. Fluoxetine (Prozac) D. Lamotrigine (Lamictal)

Correct First-line treatment for diabetic peripheral neuropathy, according to the American Diabetes Association, is tricyclic antidepressants. Anticonvulsants are second line and opioids are third line. Many medications have been found to be effective, including the tricyclics, duloxetine, pregabalin, oxycodone, and tramadol (SOR A).Among the tricyclics, amitriptyline, imipramine, and nortriptyline have been found to be the most effective (SOR A). For an uninsured patient, the tricyclics are also the most affordable.

Traveler's diarrhea can be effectively treated in the great majority of cases with which one of the following? (check one) A. Erythromycin B. Penicillin C. Sulfacetamide D. Ciprofloxacin (Cipro)

Correct Fluoroquinolones such as ciprofloxacin have been shown to significantly reduce the duration and severity of traveler's diarrhea when given for 1-3 days. Sulfacetamide is available only in a topical form for use in the eye. Penicillin and erythromycin are not effective against the most common cause of traveler's diarrhea, enterotoxigenic Escherichia coli.

A 52-year-old white male presents for a health maintenance visit. The patient has mild osteoarthritis but is otherwise healthy. He lives at home with his wife. He drinks approximately 2 beers a week and does not smoke. He takes a multivitamin, but no other medications. What is the recommendation for immunizing this patient with pneumococcal polyvalent-23 vaccine (Pneumovax)? (check one) A. One dose now B. One dose after age 65 C. One dose now, and again after age 65 D. One dose now, in 5 years, and again after age 65 E. No vaccination unless he develops an immunocompromising disease

Correct For a healthy nonsmoker with no chronic disease who is not in a high-risk group, pneumococcal vaccine is recommended once at age 65, or as soon afterward as possible. Persons that should be immunized before age 65 include patients with chronic lung disease, cardiovascular disease, diabetes mellitus, chronic liver disease, cerebrospinal fluid leaks, cochlear implants, immunocompromising conditions, or asplenia, and residents of nursing homes and long-term care facilities. The Advisory Committee on Immunization Practices of the CDC updated the recommendations for pneumococcal vaccination in 2011 to include immunization for persons age 50-64 in the following categories: Alaska Natives, Native Americans living in areas of increased risk, persons with asthma, and smokers. Ref: Centers for Disease Control and Prevention (CDC); Advisory Committee on Immunization Practices: Updated recommendations for prevention of invasive pneumococcal disease among adults using the 23-valent pneumococcal polysaccharide vaccine (PPSV23). MMWR Morb Mortal Wkly Rep 2010;59(34):1102-1105. 2) Vaughn JA, Miller RA: Update on immunizations in adults. Am Fam Physician 2011;84(9):1015-1017.

To reduce overuse of antibiotics, the CDC promotes antibiotic stewardship. The recommended intervention is the implementation of an antibiotic time-out to improve outcomes when prescribing antibiotics in hospitals. When should an antibiotic time-out be scheduled when prescribing an antibiotic at the time a patient is admitted to the hospital? (check one) A. Before starting the initial antibiotic order B. 12-24 hours after the initial antibiotic order C. 48 hours after the initial antibiotic order D. 5-7 days after the initial antibiotic order E. Prior to an antibiotic order at discharge

Correct For patients started on empiric antibiotic therapy at hospital admission, the CDC recommends an antibiotic time-out 48 hours after the initial order to determine if it can be stopped or needs to be changed. The dose, route, and duration should also be reviewed. The rationale is that antibiotics are often ordered empirically at the time of admission, while cultures and other studies are also being ordered. The original empiric order should be reassessed, incorporating the results of these studies while considering the evolving clinical status of the patient. Studies show this reassessment with antibiotic modification does not reliably occur.

The Timed Up and Go test consists of a patient rising from a chair, walking 3 meters (or about 10 feet), turning around, walking back, and sitting back down. The average healthy adult over the age of 60 can perform this in how many seconds? (check one) A. 5 B. 10 C. 20 D. 30 E. 45

Correct For the average adult over the age of 60, the normal time required for the Timed Up and Go test is 10 seconds. A time longer than 10 seconds may indicate weakness, a balance or gait problem, and/or an increased fall risk.

A 40-year-old male complains of a cough that has persisted for more than 3 months. He is otherwise asymptomatic. A chest radiograph and pulmonary tests are normal. Which one of the following is the most likely cause? (check one) A. Bronchiectasis B. Tuberculosis C. Sarcoidosos D. Asthma E. Gastroesophageal reflux disease

Correct Gastroesophageal reflux disease is one of the most common causes of chronic cough. Patients with "silent" gastroesophageal reflux may not have the classic symptoms of heartburn and regurgitation. The diagnosis is based on resolution of the cough with an empiric trial of a proton pump inhibitor, although a chest radiograph should be obtained in all patients with a chronic cough to exclude bronchiectasis, tuberculosis, and sarcoidosis. Asthma is another frequent cause of chronic cough, but it can be ruled out with normal pulmonary function tests.

The parents of a 5-year-old male ask you about treating him for attention-deficit/hyperactivity disorder (ADHD) because of his hyperactivity at home and preschool. According to the newest guidelines, the most appropriate next step is to? (check one) A. prescribe a very low dose of stimulant medication B. explain to the parents that drug therapy for ADHD is not appropriate at this age C. perform a dietary history focusing on the child's sugar intake D. explore the nature of his hyperactivity and whether there are coexisting behavioral problems

Correct Guidelines from the American Academy of Pediatrics state that stimulant medication can be prescribed for preschool children, but only after a thorough trial of behavior modification. Foods and additives have never been shown to cause or aggravate ADHD. Children with ADHD often have other behavioral problems such as depression or oppositional-defiant disorder.

A 72-year-old male is admitted to the hospital after a syncopal episode that led to a skull fracture. All of his blood tests are in the normal range. The following morning his sodium level is 132 mEq/L (N 135-145) and further testing confirms that he is suffering from the syndrome of inappropriate secretion of antidiuretic hormone (SIADH). He is placed on a free-water restriction of <1 L/day. Later that evening he complains of a headache and vomits repeatedly. A recheck of his electrolytes shows that his sodium has dropped to 121 mEq/L. What would be the most appropriate way to address his hyponatremia at this time? (check one) A. Start oral tolvaptan (Samsca) B. Start oral sodium tablets C. Start an intravenous infusion of hypertonic saline D. Further restrict fluid intake

Correct Head trauma is a known cause of the syndrome of inappropriate secretion of antidiuretic hormone (SIADH). This patient's course has been very acute, with hyponatremia developing within 48 hours. Such a precipitous drop in serum sodium may lead to cerebral and pulmonary edema. If left untreated the patient can have seizures, become obtunded, and die from brain herniation. These dangers require immediate treatment with hypertonic saline to correct the falling levels of sodium. This must be done cautiously so as to not overcorrect the sodium level too quickly, which could lead to osmotic demyelination syndrome. An increase in serum sodium levels of about 6 mEq/L should be enough to reduce symptoms and prevent progressive cerebral edema.

A 52-year-old mechanic complains of an irritation in his right eye lasting for 2 days. On direct visualization you see a small, dark foreign body on the periphery of the cornea and are able to remove it with no complications. However, there is a patch of reddish-brown discoloration extending several millimeters around the area where the foreign body had been. Which one of the following is most appropriate for this patient? (check one) A. Watchful waiting B. Irrigation with 0.9% saline solution under pressure C. An antibiotic ointment to be used every 2-4 hours D. Gentle debridement with a #11-blade scalpel E. Prompt ophthalmologic evaluation

Correct If a metal foreign body is present on the cornea for more than 24 hours a rust ring will often be present in the superficial layer of the cornea. This material is toxic to the cornea and should be removed as soon as possible, but it is not an emergency. The proper removal of a rust ring requires the use of a slit lamp and specialized ophthalmic equipment. Referral to an eye specialist within 24-48 hours is the best management in this case.

A 52-year-old male presents for a routine physical examination. His laboratory results reveal an AST (SGOT) level of 124 U/L (N 10-40) and an ALT (SGPT) level of 36 U/L (N 10-55). His (-glutamyl transpeptidase (GGTP) level is also elevated. The most likely cause of this abnormality is? (check one) A. Hepatitis B. Hemochromatosis C. Nonalcoholic fatty liver disease D. Alcoholic liver disease E. Statin-induced liver disease

Correct In a study of 256 asymptomatic Swedish patients with mildly elevated liver transaminase levels, alcohol was found to be the cause in 10% of cases. An accurate history is important for making the diagnosis. A biopsy alone cannot differentiate alcoholic liver disease from nonalcoholic fatty liver disease. An AST/ALT ratio >2 supports a diagnosis of alcoholic liver disease. Elevated (-glutamyl transpeptidase (GGTP) is also associated with alcohol abuse, especially in a patient with an AST/ALT ratio >2.

A 74-year-old male with hypertension, diabetes mellitus, and stage 3 chronic renal insufficiency presents to your office with a request from a consulting ophthalmologist for risk assessment prior to cataract surgery. His medications include lisinopril (Prinivil, Zestril), 20 mg daily; amlodipine (Norvasc), 5 mg daily; aspirin, 81 mg daily; pravastatin (Pravachol), 20 mg daily; and glipizide (Glucotrol), 5 mg daily. His blood pressure is 126/72 mm Hg. His most recent laboratory tests from 2 months ago show a hemoglobin A1c of 7.2% (N 4.0-5.6) and a serum creatinine level of 1.8 mg/dL (N 0.6-1.3). He is in his normal state of health, and is able to walk 1-2 blocks before having to stop to rest. Which one of the following would be most appropriate with regard to preoperative medical testing for this patient? (check one) A. No preoperative medical testing B. A CBC C. An EKG and cardiac stress testing D. An EKG, and if results are abnormal, stress testing or echocardiography

Correct In general, recommended preoperative testing is based on the patient's medical history and risk factors, the risk associated with the planned surgery, and the patient's functional capacity. In the case of cataract surgery, however, randomized, controlled trials have established a lack of benefit from preoperative testing for patients in their normal state of health (SOR A).

A 12-month-old male is brought to your office for a routine checkup and immunizations. He has not received medical care since his 4-month well child visit and has had no immunizations since that time.Which one of the following vaccines is NOT indicated for this patient? (check one) A. Varicella vaccine B. Rotavirus vaccine C. Hepatitis B vaccine D. MMR vaccine

Correct In general, when young children are found to be behind schedule in receiving recommended immunizations, catch-up immunization is important. However, the rotavirus series should not be started past 15 weeks of age, or continued past 8 months of age. This child should have received hepatitis B vaccine at 6 months of age, and should be given a catch-up dose. The MMR and varicella vaccines are recommended at the 12-month visit.

A 2½-year-old male is brought to the emergency department with the acute onset of diffuse abdominal pain that began approximately 6 hours ago. He has also had 3 episodes of bilious emesis in the last 2 hours. A review of systems is positive for anorexia today but negative for fever, weight loss, diarrhea, and bloody stools. On examination the patient's height and weight are in the 50th percentile for age, his blood pressure is normal, his heart rate is 110 beats/min, and his temperature is 36.9°C (98.4°F). Cardiovascular and pulmonary examinations are unremarkable. The abdominal examination is significant for slightly hypoactive bowel sounds and diffuse tenderness to palpation without rebound, guarding, or rigidity. A genitourinary examination is normal. Which one of the following studies is the most appropriate next step to diagnose the cause of abdominal pain in this patient? (check one) A. Scrotal ultrasonography B. Abdominal ultrasonography C. Abdominal and pelvic CT D. An upper gastrointestinal series

Correct In young children with bilious emesis, anorexia, and lack of fever, the most likely diagnosis is intestinal malrotation with volvulus. Abdominal ultrasonography is less sensitive and specific for malrotation than an upper gastrointestinal series, so an upper GI series should be ordered initially if volvulus is suspected. If appendicitis were suspected, ultrasonography would be preferred. CT is not a good choice because of the amount of radiation it delivers, especially given efforts to decrease the use of CT in children unless absolutely necessary. This patient's presentation is not typical for testicular torsion, therefore scrotal ultrasonography should not be the initial test of choice.

A 30-year-old male presents to your office with a 3-week history of nausea, weight loss, diarrhea, and hematochezia. He states that he has had similar episodes twice in the past and was treated at the local urgent care clinic for infectious diarrhea, with resolution of his symptoms. Your initial laboratory workup is negative for enteric pathogens and you refer the patient for colonoscopy and esophagogastroduodenoscopy with small bowel follow-through. The patient is found to have multiple noncontiguous transmural ulcerations throughout both the small and large intestines. Which one of the following initial management strategies is most likely to induce remission in this patient? (check one) A. Laparotomy with colectomy B. Metronidazole (Flagyl) C. Prednisone D. Infliximab (Remicade)

Correct Inflammatory bowel disease is divided into two categories: Crohn's disease and ulcerative colitis. Noncontiguous or "skip" lesions that are transmural in nature and are found throughout the gastrointestinal tract make a diagnosis of Crohn's disease likely in this patient. Corticosteroids are more effective in inducing remission than placebo and 5-ASA products (SOR A). A Cochrane review revealed no difference between elemental and nonelemental diets with regard to symptom remission (SOR A). Anti-TNF agents such as infliximab should be considered in patients with moderate to severe Crohn's disease who do not respond to initial corticosteroid or immunosuppressive therapy, but these are not recommended for initial treatment. While antibiotics such as metronidazole are widely used for both their anti-inflammatory and anti-infectious properties, controlled trials have not demonstrated their effectiveness. Surgical intervention should be considered in patients with ulcerative colitis, but surgery is not indicated for Crohn's disease.

Which one of the following juices can greatly increase the blood level of a statin? (check one) A. Apple B. Grapefruit C. Orange D. Pineapple E. Tomato

Correct Ingestion of grapefruit juice can increase absorption and serum levels of statins, leading to an increased risk of muscle injury. The mechanism for this is believed to be the cytochrome p-450 pathway. Starfruit juice and pomegranate juice can have a similar effect. These juices contain an irreversible inhibitor of intestinal CYP3A4, and increase the bioavailability of atorvastatin, lovastatin, and simvastatin. Rosuvastatin and fluvastatin utilize the CYP2C9 system for metabolism, so the effect on these drugs is minimal. Grapefruit juice reduces CYP3A4 activity by 50% within 4 hours of ingestion, and activity is reduced by 30% for as long as 24 hours after ingestion. Several studies document that consuming 600 mL of double-strength juice for 3 days produces a more than tenfold increase in the area under the curve for simvastatin and lovastatin, but only a 250% increase in atorvastatin.

You see a 72-year-old male for follow-up after his third hospital admission for heart failure within the past 4 months. He is a widower and lives alone, but he wants to talk about options for in-home nursing care. He is accompanied today by his sister and his neighbor. This is the first time you have seen the patient. Which one of the following is the best choice regarding your interactions with the three of them? (check one) A. Construct a family genogram to determine how to proceed with the patient's care B. Speak privately with the sister and neighbor to determine possible ulterior motives C. Determine the reason each person is present today D. Discuss the patient's health information freely, as he willingly brought the neighbor and sister along

Correct Interactions between the physician and patient involve the patient's family and friends, as well as others who may be part of the patient's social support system. These interactions, as well as the physician-patient relationship itself, are also strongly influenced by ethnic, cultural, and spiritual values and by beliefs about illness and approaches to treatment and ongoing care. Involvement of family members in a patient's care is advantageous to good communication and helpful for both accurate diagnosis and appropriate treatment (SOR C).Patient confidentiality should be protected, and the mere presence of other people accompanying the patient does not automatically constitute permission to discuss private health information with them. The physician should determine not only who is present with a patient, but also each person's reason for being present (SOR C). Speaking to relatives or friends without the patient present may be appropriate under certain circumstances but would not be the best option in this scenario. Constructing a genogram can also be helpful, but that would not be the first priority in this case.

Which one of the following organisms is the most common cause of cutaneous infections associated with intertrigo? (check one) A. Candida albicans B. Pseudomonas aeruginosa C. Staphylococcus aureus D. Group A β-hemolytic Streptococcus E. Trichophyton mentagrophytes

Correct Intertrigo is skin inflammation caused by skin-on-skin friction. It is facilitated by moisture trapped in deep skinfolds where air circulation is limited. When intertrigo does not respond to usual conservative measures, including keeping the skin clean and dry, evaluation for infection is recommended. A Wood's light examination, KOH preparation, and exudate culture can assist in identifying causative organisms. The moist, damaged skin associated with intertrigo is a fertile breeding ground for various microorganisms, and secondary cutaneous infections are commonly observed in these areas. Candida is the organism most commonly associated with intertrigo. In the interdigital spaces dermatophytes (e.g., Trichophyton rubrum, Trichophyton mentagrophytes, Epidermophyton floccosum) are more common. Staphylococcus aureus may present alone or with group A β-hemolytic Streptococcus (GABHS). Pseudomonas aeruginosa, Proteus mirabilis, or Proteus vulgaris also may occur alone or simultaneously.

A 3-year-old female is brought to your office for evaluation of mild intoeing. The child's patellae face forward, and her feet point slightly inward. Which one of the following would be most appropriate? (check one) A. Reassurance and continued observation B. Foot stretching exercises C. Orthotics D. Night splints E. Surgery

Correct Intoeing, as described in this patient, is usually caused by internal tibial torsion. This problem is believed to be caused by sleeping in the prone position and sitting on the feet. In 90% of cases internal tibial torsion gradually resolves without intervention by the age of 8. Avoiding sleeping in a prone position enhances resolution of the problem. Night splints, orthotics, and shoe wedges are ineffective. Surgery (osteotomy) has been associated with a high complication rate, and is therefore not recommended in mild cases before the age of 8.

Which one of the following medications used for anxiety has also been shown to reduce the symptoms of irritable bowel syndrome? (check one). A. Buspirone B. Clonazepam (Klonopin) C. Divalproex sodium (Depakote) D. Risperidone (Risperdal) E. Citalopram (Celexa)

Correct Irritable bowel syndrome (IBS) symptoms improve with several different medications and alternative therapies. Exercise, probiotics, antibiotics, antispasmodics, antidepressants, psychological treatments, and peppermint oil all have evidence that they may improve IBS symptoms (SOR B). A Cochrane review of 15 studies involving 922 patients found a beneficial effect from antidepressants with regard to improvement in pain and overall symptom scores compared to placebo. SSRIs used in these trials included citalopram, fluoxetine, and paroxetine, and tricyclic antidepressants included amitriptyline, desipramine, and imipramine. Buspirone, clonazepam, divalproex sodium, and risperidone have not been shown to be effective for symptom relief in IBS patients.

A 76-year-old white male with heart failure is admitted to the hospital for the third time in a year. He responds to treatment with intravenous furosemide (Lasix), oxygen, and morphine. When he is discharged, his medications include carvedilol (Coreg), 25 mg twice daily; furosemide, 40 mg daily; and lisinopril (Prinivil, Zestril), 40 mg daily. He is also placed on a low-salt diet. Which one of the following is most likely to help prevent future admissions and decrease overall medical costs for this patient during the next year? (check one) A. Case management by a heart failure specialist nurse B. Nursing home admission C. Adding amiodarone (Cordarone) D. Increasing the dosage of lisinopril

Correct Many elderly patients with heart failure are hospitalized repeatedly at short intervals. As described in a 2012 Cochrane review of 25 randomized, controlled trials, there is now good evidence that case management interventions led by a heart failure specialist nurse reduce heart failure readmissions, all-cause readmissions, and all-cause mortality. Case management interventions include home care, telephone calls, patient education, self-management, and face-to-face visits. It is not possible to say which specific interventions were optimal, but telephone follow-up was a common component in most of these trials. Multidisciplinary interventions may also be effective. For patients treated with lisinopril, 40 mg/day is the maximum amount recommended for heart failure. While $-adrenergic blockers and aldosterone antagonists are used in the treatment of heart failure, prophylactic amiodarone would not be expected to help and may be harmful. Admission to a nursing home may reduce hospitalization, but it would also increase overall costs of care.

For which one of the following respiratory infections should antibiotic therapy be initiated immediately upon diagnosis? (check one) A. Bronchitis B. Epiglottitis C. Laryngitis D. Rhinosinusitis E. Tracheitis

Correct Many infections of the respiratory tract have a viral etiology, and when this is the case early antibiotic treatment offers little to no benefit. Once the clinical course of a respiratory illness exceeds the expected length for a viral illness, it may be proper to initiate antibiotic treatment for a suspected atypical or secondary bacterial infection. Epiglottitis is one exception to this approach because of the possibility of a bacterial infection, particularly with Haemophilus influenzae type b, that can produce a rapidly worsening, potentially fatal airway compromise. When epiglottitis is suspected based on findings such as hoarseness, dysphagia, stridor, drooling, fever, chills, and respiratory distress, intravenous antibiotic treatment should be instituted immediately, ideally with a $-lactam drug that exhibits activity against methicillin-resistant Staphylococcus aureus.

You see a 16-year-old white female for a preparticipation evaluation for volleyball. She is 183 cm (72 in) tall, and her arm span is greater than her height. She wears contacts for myopia. Which one of the following should be performed at this time? (check one) A. An EKG B. Echocardiography C. A stress test D. A chest radiograph E. Coronary MR angiography

Correct Marfan syndrome is an autosomal dominant disease manifested by skeletal, ophthalmologic, and cardiovascular abnormalities. Men taller than 72 inches and women taller than 70 inches who have two or more manifestations of Marfan syndrome should be screened by echocardiography for associated cardiac abnormalities. These signs and symptoms include cardiac murmurs or clicks, kyphoscoliosis, anterior thoracic deformity, arm span greater than height, upper to lower body ratio more than 1 standard deviation below the mean, myopia, and an ectopic lens. Athletes with a family history of Marfan syndrome should also be screened, whether they have manifestations themselves or not. Patients with Marfan syndrome who have echocardiographic evidence of aortic abnormalities should be placed on β-blockers and monitored with echocardiography every 6 months.

A 46-year-old African-American female sees you because of a history of excessive uterine bleeding and irregularity in her menstrual cycle. She has three children and had a tubal ligation after her last delivery. A pelvic examination does not reveal any pathology to explain her symptoms. Further laboratory evaluation indicates that she is mildly anemic. You perform an endometrial biopsy in the office that confirms your suspicion of endometrial hyperplasia without atypia. Which one of the following is the treatment of choice for this patient? (check one) A. Elective hysterectomy B. Hysteroscopic endometrial laser ablation C. High-dose oral estrogen supplementation D. Antifibrinolytic therapy E. Progestational drugs

Correct Medical therapy with progestational drugs is the treatment of choice for menorrhagia due to endometrial hyperplasia without atypia. Progestins convert the proliferative endometrium to a secretory one, causing withdrawal bleeding and the regression of hyperplasia. The most commonly used form is cyclic oral medroxyprogesterone, given 14 days per month, but implanted intrauterine levonorgestrel is the most effective (SOR A) and also provides contraception. High-dose estrogen supplementation would further stimulate the endometrium. Estrogen is useful in cases where minimal estrogen stimulation is associated with breakthrough bleeding. The anti-fibrinolytic agent tranexamic acid prevents the activation of plasminogen and is given at the beginning of the cycle to decrease bleeding. Side effects and cost limit this treatment option, however. It may be most useful in women with bleeding disorders or with contraindications to hormonal therapy. NSAIDs, which decrease prostaglandin levels, reduce menstrual bleeding but not as effectively as progestins. While mefenamic acid is marketed for menstrual cramps and bleeding, all NSAIDs have a similar effect in this regard. If medical management fails, hysteroscopic endometrial ablation is an option for reducing uterine bleeding but is considered permanent and obviously will impair fertility. Hysterectomy is reserved for severe and chronic bleeding that is not relieved by other measures.

A 57-year-old previously healthy menopausal female presents to your office with a 1-year history of palpitations and an unintentional 10-lb weight loss. A review of systems is negative for tremors or visual changes. Vital signs include a blood pressure of 129/85 mm Hg and a heart rate of 110 beats/min. A physical examination is otherwise unremarkable except for a nontender, diffusely enlarged thyroid with no distinct nodules, and mild proptosis.Laboratory studies are significant for a TSH level<0.01 :U/mL (N 0.60-3.30), a free T3 level of 14.51 pg/mL (N 2.0-3.5), and a free T4 level of 4.52 ng/dL (N 0.71-1.40). A thyroid-stimulating immunoglobulin test is positive. In addition to a β-blocker, which one of the following is the most appropriate initial management? (check one) A. Radioactive iodine ablation B. Thyroidectomy C. Methimazole (Tapazole) D. Propylthiouracil (PTU)

Correct Methimazole and propylthiouracil (PTU) are the two oral antithyroid medications available. However, because of reports of severe hepatocellular damage, methimazole should be used instead of PTU unless it is contraindicated. Radioactive iodine treatment (131I) is an option, especially for patients who do not achieve remission with antithyroid medications. However, worsening of preexisting orbitopathy is a well-recognized potential complication of 131I treatment, as well as a transient increase in thyroid hormone levels that can precipitate thyroid storm. Thus, patients with elevated free T3 or free T4 levels should be treated with methimazole prior to 131I administration. Thyroidectomy is most often recommended for patients with thyroid nodules and those who are suspected of having cancer or who do not tolerate or refuse alternative forms of therapy. However, antithyroid medication should be given to achieve a euthyroid state prior to surgery in most patients.

A 62-year-old female presents to your office with diarrhea and signs and symptoms of dehydration. She has a temperature of 38.6°C (101.5°F) and a WBC count of 17,000/mm3 (N5300-10,800). You admit her to the hospital, and a Clostridium difficile toxin assay is positive. Because of the severity of her infection, you initiate oral vancomycin (Vancocin), 125 mg 4times daily. She has a poor clinical response and you decide to alter the antibiotic regimen to include intravenous coverage. Which one of the following intravenous antibiotics would be most appropriate? (check one) A. Ciprofloxacin (Cipro) B. Imipenem/cilastatin (Primaxin) C. Meropenem (Merrem) D. Metronidazole E. Vancomycin

Correct Metronidazole, vancomycin, and fidaxomicin are the three medications recommended for treatment of Clostridium difficile colitis infections. Only metronidazole is effective intravenously, because its biliary excretion and possibly exudation through the colonic mucosa allows it to reach the colon via the bloodstream. Treatment for this condition with vancomycin and fidaxomicin is oral. Imipenem/cilastatin, ciprofloxacin, and meropenem have not been shown to be effective for C. difficile infection.

Mild cognitive impairment is characterized by which one of the following? (check one) A. Localized motor dysfunction B. Impairment in at least one activity of daily living C. Impairment in at least one instrumental activity of daily living D. The presence of the APO E4 allele E. Objective evidence of memory decline

Correct Mild cognitive impairment is an intermediate stage between normal cognitive function and dementia. Motor function remains normal. The presence of the APO E4 allele is a risk factor, but is not necessary for a diagnosis. Patients have essentially normal functional activities but there is objective evidence of memory impairment, and the patient may express concerns about cognitive decline.

In a patient with chronic, severe, noncancer pain, which one of the following would be mostappropriate for initial opioid therapy? (check one) A. Buprenorphine (Buprenex) B. Transdermal fentanyl (Duragesic) C. Hydromorphone (Dilaudid) D. Methadone (Dolophine) E. Morphine

Correct Morphine is the best first choice for chronic potent opioid therapy (SOR B). It is reliable and inexpensive, and equivalent doses can be easily calculated if the patient must later be switched to another medication. Transdermal fentanyl and hydromorphone are reasonable second-line choices; however, they are not recommended as first-line therapy because they are expensive and can produce tolerance relatively quickly(SOR B). Methadone is another second-line option and tolerance is usually less of a problem. It is inexpensive and long-acting but also has unique pharmacokinetics. It has a very long elimination half-life, and its morphine-equivalent equianalgesic conversion ratio increases as dosages increase. Methadone can prolong the QT interval, especially in patients who are taking other QT-prolonging medications (SOR B).Buprenorphine is a partial opioid agonist that is usually used for treatment of patients with opioid addictions. Although it can be effective for treatment of pain, it is expensive and requires special prescriber training, so it is currently not recommended as a first-line agent for treatment of chronic pain (SOR C).

A 44-year-old female with localized breast cancer is receiving counseling about adjuvant long-term therapy. Which one of the following is more likely to occur with an aromatase inhibitor such as letrozole (Femara) than with tamoxifen (Soltamox)? (check one) A. Endometrial cancer B. Venous thromboembolism C. Inflammatory arthritis D. Myalgias

Correct Myalgias and noninflammatory arthralgias are more likely with aromatase inhibitors. Venous thromboembolism rarely occurs with these drugs. Endometrial cancer may occur with long-term use of tamoxifen.

A 78-year-old asymptomatic male is found to have a platelet count of 90,000/mm3 (N 150,000-300,000) and a slightly decreased WBC count. Which one of the following would be most consistent with a diagnosis of myelodysplastic syndrome? (check one) A. A normal RBC count and indices B. Normocytic anemia C. Microcytic anemia D. Macrocytic anemia E. Polycythemia

Correct Myelodysplastic syndrome is a hematologic malignancy with a predisposition to leukemic transformation. It can present with findings of anemia, thrombocytopenia, neutropenia, or any combination of these. Anemia occurs in 80%-85% of patients and is typically macrocytic.

Long-term alleviation of carpal tunnel syndrome in patients with persistent symptoms is best accomplished by which one of the following? (check one) A. Splinting B. Physical therapy C. Ibuprofen D. Corticosteroid injection E. Surgery

Correct NSAIDs, pyridoxine, and diuretics have been shown to be no more effective than placebo in the treatment of patients with carpal tunnel syndrome. Splinting, physical therapy, and corticosteroid injections have all been shown to result in short-term improvement. Patients with persistent symptoms achieve the best long-term relief with surgery.

Which one of the following is the basis for the most effective method of natural family planning? (check one) A. Calendar calculation B. Basal body temperature charting C. Cervical mucus monitoring D. Monitoring for urine estrogen metabolites E. Coitus interruptus (withdrawal)

Correct Natural family planning (NFP) is a potentially effective method for contraception and for determining the time of ovulation for purposes of conception. While the contraceptive effectiveness of the different NFP methods varies significantly, the success rates for typical use are as high as 92%-98% (SOR B). Monitoring the presence and consistency of cervical mucus production allows for the determination of both the beginning and end of a woman's most fertile period. Some NFP methods use cervical mucus secretion as the sole basis for determining fertility. The symptothermal method also incorporates calendar calculations, basal body temperature measurement, and ovulation-related symptoms as a complement to the cervical mucus component. The Marquette Model incorporates cervical mucus and basal body temperature charting with electronic monitoring of urine estrogen and LH metabolites to provide additional information to determine when ovulation has occurred.

A 35-year-old male presents with acute low back pain after he spent a weekend building a storage shed in his backyard. He has no neurologic symptoms, and the pain does not radiate into either leg. Which one of the following has been shown to be a useful treatment in this situation? (check one) A. Bed rest B. Acupuncture C. Lumbar traction D. Cyclobenzaprine (Flexeril) E. Methylprednisolone (Medrol)

Correct Nonbenzodiazepine muscle relaxants such as cyclobenzaprine are beneficial for the relief of acute low back pain for the first 7-14 days after the onset of symptoms. Patient education, physical therapy, and the application of ice or heat may also help. Unsupported treatment options for acute low back pain include oral corticosteroids, acupuncture, lumbar support, massage, chiropractic spinal manipulation, and traction. Bed rest for acute low back pain is inadvisable.

A 39-year-old male with a BMI of 41 kg/m2 is interested in weight loss. His medical history includes adequately controlled type 2 diabetes mellitus, well-controlled hypertension, hyperlipidemia, and obstructive sleep apnea. He has no history of coronary artery disease or COPD. Which one of the following is likely to be most effective for long-term weight loss in this patient? (check one) A. A very low calorie diet B. Increased physical activity C. Frequent, long-term weight-loss counseling D. Pharmacotherapy E. Bariatric surgery

Correct Obesity increases the risk of a variety of medical conditions, including type 2 diabetes mellitus, hypertension, hyperlipidemia, pulmonary disease, coronary artery disease, gallstones, fatty liver disease, obstructive sleep apnea, GERD, osteoarthritis, and a variety of forms of cancer. A weight loss of at least 10% for greater than 1 year leads to statistically significant improvement in lipid ratios, blood glucose homeostasis, and coronary artery disease risk reduction. The AAFP recommends screening for obesity and intensive counseling (more than 1 session per month for more than 3 months) with behavior modification for obese patients. Counseling is ineffective by itself and must be combined with lifestyle modification. Dietary modification, increased physical activity, and behavior modification are effective for maintaining modest weight loss for greater than 1 year (SOR B). However, there are few large, randomized, controlled trials with subjects maintaining weight reductions of 10% for over 1 year, even when combining therapy, exercise, and dietary restriction. Long-term pharmacotherapy can lead to weight loss, but regaining some weight is typical. Bariatric surgery leads to the most effective weight reduction and long-term maintenance in patients who are morbidly obese (SOR A). Gastric bypass is effective, with a mean weight loss of 71.2% at 3 years; with laparoscopic gastric banding the mean weight loss is 55.2% at 3 years. In one study, 94% of gastric bypass patients maintained at least a 20% weight loss at 6 years. Bariatric surgery has also been shown to significantly reduce fasting blood glucose, with resolution of diabetes mellitus in 31%-77% of lap band patients and 72%-100% of gastric bypass patients. Bariatric surgery is a safe and effective means for long-term weight loss and should be considered in adults with a BMI >40, or >35 with obesity-related comorbidities.

Static stretching before running has been shown to? (check one) B. Increase endurance C. Decrease the frequency of lower limb muscle injury D. Reduce delayed-onset muscle soreness E. Have no benefit

Correct Once considered generally beneficial to the running athlete, preparticipation static stretching has been found lacking in terms of benefit and even detrimental when subjected to scientific study. There is strong evidence that static stretching significantly slows performance in sprints up to 100 meters. Studies have failed to demonstrate that static stretching before running significantly decreases the likelihood of muscular injury of the lower limbs or results in a measurable reduction of delayed-onset muscle soreness. Limited evidence suggests that preparticipation static stretching, when performed alone, adversely affects both strength and endurance in elite athletes but has little measurable effect on amateur and casual athletes. Based on current understanding of sports performance, static stretching is of most benefit when performed during the cool-down period following exercise, which has been found to increase flexibility, and is best avoided immediately before athletic endeavors. A preparatory aerobic warm-up combined with dynamic range-of-motion exercises may be of some benefit for runners. Ref: Herbert RD, de Noronha M, Kamper SJ: Stretching to prevent or reduce muscle soreness after exercise. Cochrane Database Syst Rev 2011;(7):CD004577. 2) Yeung SS, Yeung EW, Gillespie LD: Interventions for preventing lower limb soft-tissuerunning injuries. Cochrane Database Syst Rev 2011;(7):CD001256.

You suspect orthostatic hypotension in an elderly male who reports "dizziness" when standing up, and you decide to obtain recumbent and standing blood pressure measurements. After the patient rests in a supine position for 5 minutes, you measure his baseline blood pressure and then ask him to stand, which he does without a problem. For how long should his blood pressure be periodically measured before considering the test complete? (check one) A. 30 seconds B. 60 seconds C. 90 seconds D. 3 minutes E. 5 minutes

Correct Orthostatic hypotension is defined as a documented drop in blood pressure of at least 20 mm Hg systolic or 10 mm Hg diastolic that occurs within 3 minutes of standing. When symptomatic it is often described as lightheadedness or dizziness upon standing. Etiologies to consider include iatrogenic, neurologic, cardiac, and environmental causes, plus many others alone or in combination. Since orthostatic hypotension may result in syncope, leading to falls and substantial injury, identifying it and taking corrective steps can produce a significant benefit.

The diagnosis of Osgood-Schlatter disease (osteochondritis of the tibial tubercle apophysis) is best made on the basis of findings from: (check one) A. the history and examination B. evaluation by an orthopedic specialist C. radiographs D. ultrasonography E. MRI

Correct Osgood-Schlatter disease is an inflammatory condition that is a common cause of knee pain in children and adolescents. The diagnosis is usually based on clinical findings, although radiographs may be necessary to rule out fractures or other problems if findings are not typical. MRI, ultrasonography, and orthopedic referral are not usually needed. The problem is typically self-limited and responds to activity modification, over-the-counter analgesics, stretching, and physical therapy.

Which one of the following is the most accurate imaging study for assessing early osteomyelitis? (check one) A. Plain radiography B. Ultrasonography C. CT D. MRI E. A bone scan

Correct Osteomyelitis is a serious complication of diabetic foot infections and is present in up to 20% of mild to moderate infections and in 50%-60% of severe infections. While a bone biopsy and/or bone cultures are definitive for making the diagnosis, radiologic studies can also be helpful. Plain radiography may show bony destruction but has a sensitivity for osteomyelitis ranging from 28% to 75%, depending on the timing of the examination and the severity of the infection. It may take weeks for these infections to become apparent on plain radiographs. The sensitivity of triple-phase technetium bone scans is up to 90% but they have low specificity for osteomyelitis. The 90% sensitivity and 80% specificity of MRI is superior to all other imaging modalities.

Human parvovirus B19 is associated with which one of the following? (check one) A. Erythema marginatum B. Erythema multiforme C. Erythema toxicum D. Erythema infectiosum E. Erythema chronicum

Correct Parvovirus B19 is associated with erythema infectiosum, or fifth disease. It is also associated with nonspecific fever, arthropathy, chronic anemia, and transient aplastic crisis.

A 78-year-old male is on dual antiplatelet therapy (aspirin and clopidogrel) as a result of a stroke 6 months ago. He recently underwent coronary angiography, and his cardiologist has scheduled coronary artery bypass surgery for a week from today. Which one of the following is recommended with regard to his antiplatelet therapy? (check one) A. Stopping only aspirin 5 days before surgery B. Stopping only clopidogrel 5 days before surgery C. Stopping both aspirin and clopidogrel 5 days before surgery D. Continuing both aspirin and clopidogrel

Correct Patients receiving dual antiplatelet therapy who require bypass surgery should continue taking aspirin. Clopidogrel or prasugrel should be stopped 5 days before the surgery due to the increased risk of major bleeding during surgery.

A 66-year-old female sees you for the first time. She has a history of iron deficiency anemia and chronic diarrhea associated with a diagnosis of celiac disease. This history increases her risk for which one of the following? (check one) A. Diverticulitis B. Ulcerative colitis C. Crohn's disease D. Colon cancer E. Osteoporosis

Correct Patients who are diagnosed with celiac disease are at increased risk of osteoporosis due to bone loss from decreased calcium and vitamin D absorption. These patients are at higher risk for fractures. Patients with celiac disease are not at increased risk for inflammatory bowel disease, diverticulitis, or colon cancer.

According to national and international guidelines, which one of the following is the next step for adults with asthma who require therapy with inhaled β-agonists more than three times a week? (check one) A. Inhaled glucocorticoids B. Inhaled salmeterol (Serevent) C. Sustained-release oral β-agonists D. Sustained-release oral theophylline

Correct Patients who require inhalation therapy with β2-adrenergic-receptor agonists more than twice weekly but not daily have mild persistent asthma. Long-term control with inhaled corticosteroids is recommended for adults with persistent asthma.

A 55-year-old male with a 4-year history of type 2 diabetes mellitus was noted to have microalbuminuria 6 months ago, and returns for a follow-up visit. He has been on an ACE inhibitor and his blood pressure is 140/90 mm Hg.The addition of which one of the following medications would INCREASE the likelihood that dialysis would become necessary? (check one) A. Hydrochlorothiazide B. Amlodipine (Norvasc) C. Atenolol (Tenormin) D. Clonidine (Catapres) E. Losartan (Cozaar)

Correct Patients with diabetes mellitus, atherosclerosis, and end-organ damage benefit from ACE inhibitors and angiotensin receptor blockers (ARBs) equally when they are used to prevent progression of diabetic nephropathy. Combining an ACE inhibitor with an ARB is not recommended, as it provides no additional benefit and leads to higher creatinine levels, along with an increased likelihood that dialysis will become necessary.

Which one of the following medications used to treat psychiatric disorders is associated with an increased risk of agranulocytosis? (check one) A. Carbamazepine (Tegretol) B. Lithium C. Aripiprazole (Abilify) D. Olanzapine (Zyprexa) E. Imipramine (Tofranil)

Correct People taking carbamazepine have a five-to eightfold increased risk of developing agranulocytosis. Baseline values including a CBC, serum electrolytes, and liver enzymes should be obtained before the drug is started, and the patient should be monitored with periodic hematologic testing. The other medications listed are not associated with agranulocytosis. Aripiprazole and olanzapine carry black box warnings for an increased risk of death in the elderly. Lithium is associated with lithium toxicity and thyroid dysfunction. Imipramine carries a warning for cardiac toxicity, and EKG monitoring is recommended.

A 7-year-old male presents with a 3-day history of sore throat, hoarseness, fever to 100°F (38°C), and cough. Your examination reveals injection of his tonsils, no exudates, shotty lymphadenopathy, and normal breath sounds. Which one of the following would be most appropriate? (check one) A. Symptomatic treatment only B. Empiric treatment for streptococcal pharyngitis C. A rapid antigen test for streptococcal pharyngitis D. A throat culture for streptococcal pharyngitis E. An office test for mononucleosis

Correct Pharyngitis is a common complaint, and usually has a viral cause. The key factors in diagnosing streptococcal pharyngitis are a fever over 100.4°F, tonsillar exudates, anterior cervical lymphadenopathy, and absence of cough. The scenario described is consistent with a viral infection, with no risk factors to make streptococcal infection likely; therefore, this patient should be offered symptomatic treatment only. Testing for other infections is not indicated unless the patient worsens or does not improve.

A 22-year-old male who is training for a marathon presents with a 2-week history of ankle pain. There is no history of trauma to the ankle. Your examination reveals tenderness over the distal tibia and you suspect a stress fracture. The initial imaging study of choice for this patient's ankle is? (check one) A. A plain radiograph B. Ultrasonography C. CT D. MRI E. A radionuclide bone scan

Correct Plain radiography should be the initial imaging modality to diagnose stress fractures (SOR C). One algorithm advocates radiography 2 weeks after the onset of symptoms (if symptoms persist), with repeat radiography the following week before performing more advanced imaging. An expert panel of the American College of Radiology recommends that MRI be considered next if plain radiography is negative.

Which one of the following is most suggestive of plantar fasciitis? (check one) A. Heel pain at rest B. A heel spur on radiographs C. Prompt relief with NSAIDs D. Heel pain that is worse with the first steps in the morning

Correct Plantar fasciitis is characterized by pain that is worse with the first few steps in the morning or after a prolonged rest. NSAIDs may help with the discomfort, but prompt relief of the pain by any modality is not common. The pain is typically in the medial heel. While 50% of people with plantar fasciitis have heel spurs on radiographs, this finding is not causative or diagnostic. The diagnosis is made clinically.

A 48-year-old female presents as a new patient to your office. She has not seen a physician for several years and her medical history is unknown. Her BMI is 24.4 kg/m2 and she is not taking any medication. Her blood pressure is 172/110 mm Hg in the left arm sitting and 176/114 mm Hg in the right arm sitting; her cardiovascular examination is otherwise unremarkable. A baseline metabolic panel reveals a creatinine level of 0.68 mg/dL (N 0.6-1.1) and a potassium level of 3.3 mEq/L (N 3.5-5.5). If the patient's hypertension should prove refractory to treatment, which one of the following tests is most likely to reveal the cause of her secondary hypertension? (check one) A. A 24-hour urine catecholamine level B. A plasma aldosterone/renin ratio C. MRA of the renal arteries D. Echocardiography E. A sleep study (polysomnography)

Correct Primary hyperaldosteronism is the most common cause of secondary hypertension in the middle-aged population, and can be diagnosed from a renin/aldosterone ratio. This diagnosis is further suggested by the finding of hypokalemia, which suggests hyperaldosteronism even though it is not present in the majority of cases. An echocardiogram would help make a diagnosis of coarctation of the aorta, but this is more common in younger patients. Renal MRA may demonstrate renal artery stenosis, but this condition is more common in older patients. Sleep apnea is increasing in prevalence along with the rise in obesity, but it is not suggested by this case. A 24-hour urine catecholamine test is used to diagnose pheochromocytoma, which is not suggested by this patient's findings. Pheochromocytoma is also less common than aldosteronism (SOR C).

A 50-year-old female reports a 1-month history of pain in her wrists. She does not recall any injury. On examination both wrists are warm but not red, feel boggy on palpation, and lack 30° of both flexion and extension. No other joints are affected. She feels fatigued and unwell, but attributes this to her busy schedule. Radiographs of the wrists are normal. Laboratory findings are unremarkable except for a mildly elevated erythrocyte sedimentation rate and a negative rheumatoid factor. Which one of the following is the most likely diagnosis? (check one) A. Rheumatoid arthritis B. Osteoarthritis C. Inapparent injury D. Fibromyalgia E. Lyme disease

Correct Rheumatoid arthritis is most often symmetric at presentation and particularly affects the wrists and other extremity joints that have a high ratio of synovium to articular cartilage. Rheumatoid factor is often negative in the early months of the disease, although it may be positive later. Radiographs and laboratory tests are helpful, but the diagnosis is primarily clinical. Osteoarthritis of the wrists usually involves the carpal-metacarpal joint of the thumb primarily, and the joint would be red if there were an injury. Fibromyalgia usually involves the soft tissue of the trunk, and there is no evidence of inflammation. Lyme disease can cause a variety of joint diseases, but not chronic symmetric arthritis.

An 11-year-old female is brought to your office for a routine well child examination. The American Academy of Pediatrics recommends screening this patient for which one of the following? (check one) A. Anemia B. Diabetes mellitus C. Dyslipidemia D. HIV

Correct S Because of concerns about the growing epidemic of obesity in this population, the American Academy of Pediatrics now recommends screening for elevated serum cholesterol levels in children 9-11 years of age (SOR C). This patient should also be screened annually for depression beginning at 11 years of age and continuing through 21 years of age. Universal screening for iron deficiency anemia is recommended at 12 months of age and again at 15-30 months of age if the patient is determined to be high risk. HIV screening is recommended in adolescents 16-18 years of age, and age 21 is now the recommended starting age for screening for cervical dysplasia. Universal screening for diabetes mellitus is not recommended for children or adolescents.

Which one of the following classes of diabetes medications increases the risk of genitourinary infections by blocking glucose reabsorption by the kidneys? (check one) A. SGLT2 inhibitors such as canagliflozin (Invokana) B. GLP-1 receptor agonists such as exenatide (Byetta) C. DPP-4 inhibitors such as sitagliptin (Januvia) D. Prednisone, 2-3 mg/kg daily E. Meglitinides such as repaglinide (Prandin)

Correct SGLT2 inhibitors inhibit SGLT2 in the proximal nephron. This blocks glucose reabsorption by the kidney, increasing glucosuria. The advantages of this medication include no hypoglycemia, decreased weight, decreased blood pressure, and effectiveness at all stages of type 2 diabetes mellitus. Disadvantages are that it increases the risk of genitourinary infections, polyuria, and volume depletion and increases LDL-cholesterol and creatinine levels. GLP-1 receptor agonists work by activating the GLP-1 receptors, causing an increase in insulin secretion, a decrease in glucagon secretion, slowing of gastric emptying, and increasing satiety. DPP-4 inhibitors inhibit DPP-4 activity, which increases postprandial active incretin concentration. This increases insulin secretion and decreases glucagon secretion. Meglitinides act by closing the ATP-sensitive K+ channels on the B-cell plasma membranes, which increases insulin secretion."-Glucosidase inhibitors inhibit intestinal "-glucosidase, which slows intestinal carbohydrate digestion and absorption.

A mother meets you in the emergency department with her 3-week-old infant. The infant was delivered at term, with an uneventful prenatal and postnatal course to this point. The mother reports that the infant stopped breathing for 20-25 seconds, and that his lips and tongue appeared bluish. There was no coughing, choking, or congestion, but the child seemed "limp." The episode ended when the mother vigorously stimulated her child and he started crying. On examination, the child appears normal. Which one of the following would be most appropriate at this point? (check one) A. Reassurance and no further evaluation B. Discharge with a home apnea monitor C. Hospital admission for observation D. Mandatory referral to child protective services E. Direct laryngoscopy to rule out a foreign body

Correct Some experts recommend inpatient observation for all children with apparent life-threatening events such as this. Hospital admission is not always necessary, however, for a short, self-correcting episode associated with feeding. Given the history of not breathing for 20-25 seconds, having a blue tongue and lips, and being limp, admitting the child for observation is appropriate. Although child abuse is a concern, referral to child protective services is not mandatory. Laryngoscopy would not be routine, but might be appropriate in some cases depending on the history and physical findings. Many groups recommend home apnea monitoring after discharge for patients with more severe or undiagnosed cases.

A 45-year-old female is being treated for hypothyroidism with levothyroxine (Synthroid), 112 :g daily. She is still having persistent fatigue and weight gain despite her TSH value of 1.5 :U/mL (N 0.5-5.5). In addition to evaluating this patient for other causes of her symptoms, which one of the following would be appropriate management of her thyroid medication at this time? (check one) A. Continuing the current therapy B. Increasing the dosage C. Adding liothyronine (Cytomel) D. Switching to desiccated thyroid hormone (Armour Thyroid)

Correct Some hypothyroid patients who are treated with appropriate dosages of levothyroxine and whose TSH levels are in the appropriate range continue to have persistent symptoms such as fatigue, depressed mood, and weight gain. If the TSH is in the appropriate range then no adjustment is necessary and annual serum TSH testing is recommended. Patients who remain symptomatic on an appropriate dosage of levothyroxine, as determined by a TSH <2.5 mIU/L, are not likely to benefit from combination triiodothyronine/thyroxine therapy (SOR A). Desiccated thyroid hormone preparations are not recommended by the American Association of Clinical Endocrinologists for the treatment of hypothyroidism. A meta-analysis of 11 randomized, controlled trials of combination T3/T4 therapy versus T4 monotherapy showed no improvements in pain, depression, or quality of life (SOR A).

A 54-year-old male sees you for a 6-month follow-up visit for hypertension. He feels well, but despite the fact that he takes his medications faithfully, his blood pressure averages 150/90 mm Hg. He has had an intensive workup for hypertension in the recent past, with normal repeat laboratory results, including a CBC, serum creatinine, an electrolyte panel, and a urinalysis. His medications include chlorthalidone, 12.5 mg daily; carvedilol (Coreg), 25 mg twice daily; amlodipine (Norvasc), 10 mg daily; and lisinopril (Prinivil, Zestril), 40 mg daily. He has been intolerant to clonidine (Catapres) in the past. Which one of the following medication changes would be most reasonable? (check one) A. Adding isosorbide mononitrate (Imdur) B. Adding spironolactone (Aldactone) C. Substituting furosemide (Lasix) for chlorthalidone D. Substituting losartan (Cozaar) for lisinopri

Correct Spironolactone is now recommended for treating resistant hypertension, even when hyperaldosteronism is not present. A longer-acting diuretic such as chlorthalidone is also recommended for treating hypertension, particularly in resistant cases with normal renal function. There is no benefit to switching from an ACE inhibitor to an ARB. Nitrates have some effect on blood pressure but are recommended only for patients with coronary artery disease.

A 26-year-old male presents to the emergency department with a fever, and he appears acutely ill. After a previously undocumented grade 3 murmur is detected on examination, a transthoracic echocardiogram is ordered and reveals a 1.5-cm vegetation on the tricuspid valve. Which one of the following is the most likely causative organism? (check one) A. Cardiobacterium hominis B. Enterococcus faecalis C. Pseudomonas aeruginosa D. Staphylococcus aureus E. Streptococcus viridans

Correct Staphylococcus aureus is the most common cause of acute infectious endocarditis worldwide. Additionally, the most common cause of tricuspid valve endocarditis is intravenous drug abuse, and Staphylococcus aureus is the infecting organism in 80% of tricuspid valve infections. Streptococcus viridans is also a frequent cause of infectious endocarditis, with Enterococcus, Pseudomonas, and Cardiobacterium being less likely causes.

A 75-year-old male with a history of hypertension sees you after experiencing an episode of numbness on his right side and loss of strength in his right arm. The numbness and weakness resolved spontaneously within 20 minutes. Carotid Doppler ultrasonography and cerebral angiography both reveal significant carotid stenosis. In addition to starting aspirin, which one of the following would be the most appropriate next step for this patient? (check one) A. Aggressive lowering of blood pressure B. Clopidogrel (Plavix) C. Carotid artery stenting D. Evaluation for occult patent ductus arteriosus E. High-dose statin therapy

Correct Statin drugs are effective for preventing stroke, which should be the key goal in this high-risk patient. They may stabilize the intimal wall. Rapid lowering of blood pressure could cause brain injury by reducing blood flow in patients with carotid stenosis. Any evidence of hypoperfusion needs to be corrected immediately. Combination therapy with aspirin and clopidogrel is associated with an increased risk of bleeding and is not recommended for stroke prevention. Patients over age 70 have worse outcomes with carotid stenting than with endarterectomy. Occult patent ductus arteriosus has not been shown to be a significant risk factor for stroke.

A 67-year-old male is admitted to your inpatient service with a week-long acute exacerbation of COPD. He also has hypertension and type 2 diabetes mellitus. After 24 hours of intravenous fluids and intravenous methylprednisolone, he is now tolerating oral intake. Which one of the following corticosteroid regimens is best for this patient at this time? (check one) A. Continue intravenous methylprednisolone until his COPD is back to baseline, then switch to oral methylprednisolone for a 14-day total course of treatment B. Switch to oral prednisone for a 14-day total course of treatment, including the initial 24-hour intravenous treatment C. Switch to oral prednisone for 4 more days of treatment D. Use only inhaled corticosteroids by nebulizer E. Discontinue corticosteroid treatment altogether after 24 hours

Correct Systemic corticosteroid therapy reduces the hospital length of stay in patients with acute COPD exacerbations (SOR A). Oral therapy has been shown to be as effective as the intravenous route in patients who can tolerate oral intake (SOR B). A randomized, controlled trial has demonstrated that 5-day courses of systemic corticosteroid therapy are at least as effective as 14-day courses (SOR A). Inhaled corticosteroids are beneficial in some COPD patients but nebulizers generally do not offer significant advantages over metered-dose inhalers in most patients.

To prevent joint damage from gout, uric acid levels should be lowered by medication to (check one) A. <6.0 mg/dL B. <8.0 mg/dL C. <10.0 mg/dL D. a level that keeps the patient symptom-free for 6 months

Correct Targets for uric acid levels in patients with gout vary according to published guidelines but range from 5 to 6 mg/dL. Patients may be symptom-free at higher levels but risk joint damage even without acute episodes (SOR A).

According to the American Diabetes Association, screening for diabetes mellitus in the asymptomatic patient with no risk factors should begin at which age? (check one) A. 25 years B. 30 years C. 35 years D. 40 years E. 45 years

Correct Testing for diabetes mellitus should be considered in all asymptomatic adults who have a BMI ≥25 kg/m2 and have one or more additional risk factors such as physical inactivity, a first degree relative with diabetes, a high-risk ethnicity, hypertension, hyperlipidemia, or polycystic ovary syndrome. In asymptomatic patients with no risk factors, screening should begin at age 45. Ref: Ismail-Beigi F: Glycemic management of type 2 diabetes mellitus. N Engl J Med 2012;366(14):1319-1327. 2) American Diabetes Association: Standards of medical care in diabetes—2013. Diabetes Care 2013;36(Suppl 1):S13.

A large wooden splinter went deep into the forearm of a 24-year-old male while he was working in a horse barn, and he has required local anesthesia and a small incision to remove it completely. After thorough wound cleansing, you inquire about his tetanus status. He is certain that he received all of his primary childhood vaccines and a "tetanus booster" at age 20, but does not know which vaccine he received. Which one of the following is the best choice for this patient regarding tetanus immunization at this time? (check one) A. TT (tetanus toxoid) B. Td (tetanus toxoid with reduced diphtheria C. Tdap (tetanus toxoid with reduced diphtheria and acellular pertussis) D. TIG (tetanus immune globulin E. No immunization

Correct The Advisory Committee on Immunization Practices (ACIP) periodically makes recommendations for routine or postexposure immunization for a number of preventable diseases, including tetanus. Since 2005, the recommendation for tetanus prophylaxis has included coverage not only for diphtheria (Td) but also pertussis, due to waning immunity in the general population. The current recommendation for adults who require a tetanus booster (either as a routine vaccination or as part of treatment for a wound) is to use the pertussis-containing Tdap unless it has been less than 5 years since the last booster in someone who has completed the primary vaccination series. In this scenario, no additional vaccination is needed at this time, since the patient is certain of completing the primary vaccinations and received a tetanus booster within the previous 5 years. Had the interval been longer than 5 years, then a single dose of Tdap would be appropriate unless his previous booster was Tdap. Tetanus immune globulin is recommended in addition to tetanus vaccine for wounds that are tetanus-prone due to contamination and tissue damage in persons with an uncertain primary vaccine history. Plain tetanus toxoid (TT) is usually indicated only when the diphtheria component is contraindicated, which is uncommon.

The mother of a 6-month-old male tells you that he sometimes wheezes while feeding, and this is occasionally associated with a cough. Changing his position does not help. Which one of the following is the most likely diagnosis? (check one) A. Tracheoesophageal fistula B. Laryngeal cleft C. Gastroesophageal reflux disease D. Foreign body aspiration E. Tracheomalacia

Correct There are many causes of wheezing in infants and children. Wheezing associated with feeding is most commonly due to gastroesophageal reflux disease (level of evidence 3). Tracheoesophageal fistula and laryngeal cleft also cause wheezing associated with feeding, but are rare. Foreign body aspiration is most common between 8 months and 4 years of age and the child is most likely to have a history of the sudden onset of wheezing associated with choking. The wheezing present with tracheomalacia is position related.

A 52-year-old male has a skin lesion removed from his arm with appropriate sterile precautions. Which one of the following would be most appropriate to use on this surgical wound? (check one) A. Petrolatum B. Silver sulfadiazine (Silvadene) cream C. Mupirocin (Bactroban) ointment D. Polymyxin B/bacitracin ointment (Polysporin) E. Triple-antibiotic (neomycin/polymyxin B/bacitracin) ointment

Correct The American Academy of Dermatology recommends against the routine use of topical antibiotics for clean surgical wounds, based on randomized, controlled trials. Topical antibiotics have not been shown to reduce the rate of infection in clean surgical wounds compared to the use of nonantibiotic ointment or no ointment. Studies have shown that white petrolatum ointment is as effective as antibiotic ointment in postprocedure care. Topical antibiotics can aggravate open wounds, hindering the normal wound-healing process. In addition, there is a significant risk of developing contact dermatitis, as well as a potential for antibiotic resistance. Antibiotic treatment should be reserved for wounds that show signs of infection.

A healthy 24-year-old male presents with a sore throat of 2 days' duration. He reports mild congestion and a dry cough. On examination his temperature is 37.2°C (99.0°F). His pharynx is red without exudates, and there are no anterior cervical nodes. His tympanic membranes are normal, and his chest is clear. Which one of the following would be most appropriate at this point? (check one) A. Analgesics and supportive care only B. A rapid strep test C. A throat culture and empiric treatment with penicillin D. Azithromycin (Zithromax)

Correct The Centers for Disease Control and Prevention (CDC) assembled a panel of national health experts to develop evidence-based guidelines for evaluating and treating adults with acute respiratory disease. According to these guidelines, the most reliable clinical predictors of streptococcal pharyngitis are the Centor criteria. These include tonsillar exudates, tender anterior cervical lymphadenopathy, absence of cough, and history of fever. The presence of three or four of these criteria has a positive predictive value of 40%-60%, and the absence of three or four of these criteria has a negative predictive value of 80%. Patients with four positive criteria should be treated with antibiotics, those with three positive criteria should be tested and treated if positive, and those with 0-1 positive criteria should be treated with analgesics and supportive care only. This patient has only one of the Centor criteria, and should therefore not be tested or treated with antibiotics.

An 11-year-old male is brought to your clinic for follow-up after a recent well child visit revealed elevated blood pressure. The parents have restricted his intake of sodium and fatty foods during the last several weeks. His blood pressure today is 140/92 mm Hg, which is similar to the reading at his last visit. The parents checked the child's blood pressure with a home unit several times and found it consistently to be in the 130s systolic and low 80s diastolic. The child had a normal birth history and has no known chronic medical conditions. Both of his parents and his two younger siblings are healthy. He is at the 75th percentile for both height and weight with a BMI in the normal range. He eats a balanced diet and is active. What should be the next step for this patient? (check one) A. Reassurance that this is likely white-coat hypertension B. A goal weight loss of at least 5 lb C. Evaluation for causes of secondary hypertension D. Hydrochlorothiazide E. Lisinopril (Prinivil, Zestril)

Correct The Fourth Report on the Diagnosis, Evaluation, and Treatment of High Blood Pressure in Children and Adolescents defines hypertension in children as a systolic or diastolic blood pressure above the 95th percentile for the patient's sex, age, and height on several different readings. Although it is appropriate to have this finding confirmed in the outpatient setting, 130 mm Hg is still at the 99th percentile for systolic blood pressures in this patient. Hypertension in a patient this young should prompt a search for secondary causes, which are more common in young hypertensive patients than in adults with hypertension. The recommended workup includes blood and urine testing, as well as renal ultrasonography. An evaluation for end-organ damage is also recommended, including retinal evaluation and echocardiography.

The Infectious Diseases Society of America recommends which one of the following as the drug of choice for group A streptococcal pharyngitis? (check one) A. Azithromycin (Zithromax) B. Cefadroxil C. Cephalexin (Keflex) D. Clindamycin (Cleocin) E. Penicillin

Correct The Infectious Diseases Society of America recommends that penicillin remain the treatment of choice for group A streptococcal pharyngitis because of its proven efficacy, safety, narrow spectrum, and low cost. Penicillin-resistant group A Streptococcus has never been documented. Amoxicillin is often used in place of penicillin V as oral therapy for young children, primarily because of acceptance of the taste of the suspension. The other options listed are all possible regimens for group A streptococcal pharyngitis but penicillin is still considered the treatment of choice.

A 62-year-old male with a 20-year history of diabetes mellitus presents with bilateral calf and buttock pain that occurs after he walks 2 blocks. The symptoms are relieved with rest. On examination his pedal pulses are not palpable and his ankle-brachial index is 1.45. Which one of the following would be most appropriate? (check one) A. Reassuring the patient that his ankle-brachial index is normal B. MRI of the lumbar spine C. A repeat evaluation in 6 months if the symptoms persist D. MR or CT angiography of the lower extremities

Correct The National Health and Nutrition Examination Survey (NHANES) found that 1.4% of adults over 40 have an ankle-brachial index (ABI) >1.4; this group accounts for approximately 20% of all adults with peripheral artery disease. An ABI >1.4 indicates noncompressible arteries (calcified vessels). In patients with arterial calcification, such as diabetic patients, more reliable information is often obtained by using toe pressures to calculate a toe-brachial index and from pulse volume recordings. Vascular imaging should be used to confirm peripheral vascular disease. MR or CT arteriography, duplex scanning, and hemodynamic localization are noninvasive methods for lesion localization and may be helpful when symptoms or findings do not correlate with the ABI. Contrast arteriography is used for definitive localization before intervention.

A 70-year-old white female asks you to evaluate her right shoulder because of pain and limited range of motion. Further history reveals that 2 months ago she slipped in her kitchen and caught onto the refrigerator door handle to avoid falling to the floor. On examination she has pain and weakness at 45° of abduction and weakness on external rotation. She should be treated for? (check one) A. Bicipital tendinitis B. Disruption of the glenoid fossa C. Rotator cuff tear D. Acromioclavicular separation E. Incomplete fracture of the humeral head

Correct This patient has a history and physical findings that are consistent with a rotator cuff tear. Most commonly the mechanism of injury in an acute rotator cuff tear is forced abduction of the arm with significant resistance. Often this will occur when a person attempts to break a fall with an outstretched hand. There is usually a sudden sensation of tearing pain in the shoulder. Pain and muscle spasm will limit shoulder motion. Patients with a large tear cannot initiate shoulder abduction and will have a discrepancy between active and passive motion. Patients with significant tears will also have a positive drop arm test. This test is performed by passively abducting the arm to 90° and asking the patient to hold the arm in that position while the examiner applies pressure on the distal forearm or wrist. The test is positive if the pressure causes the arm to drop suddenly. Acute tears are generally managed with a splint and orthopedic referral for surgical repair. Chronic tears may be managed with shoulder rehabilitation but may ultimately require surgical repair as well. Bicipital tendinitis is not generally caused by acute trauma, but by irritation and microtrauma due to repetitive elevation or abduction of the shoulder, causing an inflammatory reaction in the synovial sheath. Patients generally present with a complaint of pain in the anterior shoulder that radiates into the upper arm. It is more painful with activity and is worse at night. Abduction and external rotation of the arm exacerbates the pain. On examination there should be point tenderness in the bicipital groove. Active range of motion will be limited by pain but passive range of motion will be intact. There should not be any weakness. Acromioclavicular separation is usually caused by a fall or a direct blow to the point of the shoulder with the shoulder abducted. The pain associated with this injury is over the acromioclavicular joint margin and there may be swelling. Depending on the severity of the injury there may be full range of motion but it may be restricted due to pain. There should not be any weakness associated with this injury. A fracture of the humeral head generally occurs with a fall on an outstretched arm or direct blow to the lateral side of the arm. Generally there is pain or bruising over the fracture site. Movement will be restricted by pain, but there should not be any weakness. A tear of the labrum can occur with acute trauma or from repetitive shoulder motion. Acute trauma may occur from a dislocation of the shoulder, falling on an outstretched arm, or direct blows to the shoulder. Generally, people with a tear of the labrum will have increased pain with overhead activity, popping or grinding, loss of strength, and trouble locating a specific point of pain.

An 18-year-old female presents with a painful right ankle after twisting it during a basketball game. On examination she has no tenderness over the lateral malleolus or posterior distal fibula, which she has identified as the location of the pain. According to the Ottawa ankle rules, which one of the following would indicate that an ankle radiograph should be performed? (check one) A. Moderate ankle swelling B. The inability to bear weight on the right foot with the left foot elevated for 5 seconds C. The inability to stand for 5 seconds with weight evenly distributed on both feet, at the time of injury and during the evaluation D. The inability to take 4 steps at the time of the injury and during the evaluation E. The inability to walk 6 feet during the evaluation

Correct The Ottawa ankle rules are 99% sensitive and 58% specific for identifying a fracture. They state that ankle radiography should be performed when a patient presents with pain in the malleolar region and has either point tenderness over the tip of the malleolus or the posterior edge of the affected bone (distal 6 cm), or is unable to bear weight at the time of injury and while being evaluated in the emergency department or office. Inability to bear weight is defined as the inability to take four steps. A limp when weight is transferred to the affected extremity still counts as being able to bear weight.

A 75-year-old female is evaluated in the emergency department in the evening for heart failure. She is acutely symptomatic with dyspnea. Vital signs include a pulse rate of 96 beats/min, a blood pressure of 140/90 mm Hg, and an oxygen saturation of 94% on room air. A chest radiograph shows mild pulmonary congestion. Which one of the following would be most appropriate regarding placement of an indwelling urinary catheter for accurate measurement of urine output and for patient comfort? (check one) A. Avoiding indwelling urinary catheter placement B. Placement of an indwelling urinary catheter only until initial diuresis is completed C. Placement of an indwelling urinary catheter and removal when the patient is transferred out of the emergency department D. Placement of an indwelling urinary catheter until 6:00 a.m. tomorrow E. Placement of an indwelling urinary catheter and removal within 24 hours

Correct The Society of Hospital Medicine recommends that urinary catheters not be placed or left in place for managing incontinence or for staff convenience, or for monitoring output in patients who are not critically ill. The Infectious Diseases Society of America recommends using patient weight to monitor diuresis. Acceptable indications for an indwelling catheter include critical illness, obstruction, hospice care, and perioperatively for <2 days for urologic procedures.

A 25-year-old white male truck driver presents with a 1-day history of throbbing rectal pain. Your examination shows a large thrombosed external hemorrhoid. Which one of the following is the preferred initial treatment for this patient? (check one) A. Infrared coagulation B. Rubber band ligation of the hemorrhoid C. Elliptical excision of the thrombosed hemorrhoid D. Stool softeners and a topical analgesic/hydrocortisone cream

Correct The appropriate management of a thrombosed hemorrhoid presenting within 72 hours of the onset of symptoms is elliptical excision of the hemorrhoid and overlying skin under local anesthesia, such as 0.5% bupivacaine hydrochloride in 1:200,000 epinephrine, infiltrated slowly with a 27-gauge needle. Incision and clot removal may provide inadequate drainage, resulting in rehemorrhage and clot reaccumulation. Most thrombosed hemorrhoids contain multilocular clots that may not be accessible through a simple incision. Rubber band ligation is an excellent technique for management of internal hemorrhoids, and infrared coagulation is also used for this purpose. Banding an external hemorrhoid would cause exquisite pain. If the pain is already subsiding or more time has elapsed, and if there is no necrosis or ulceration, measures such as sitz baths, bulk laxatives, stool softeners, and local analgesia may be helpful. Some local anesthetics carry the risk of sensitization. Counseling to avoid precipitating factors such as prolonged standing/sitting, constipation, and delay of defecation is also appropriate.

A 2-year-old male is brought in for an initial office visit. He just moved to your community to live with foster parents. On examination, you note a thin upper lip, a smooth philtrum, a flat nasal bridge, small palpebral fissures, a curved fifth finger (clinodactyly), and a widened upper palmar crease that ends between the second and third fingers. These findings suggest which one of the following? (check one) A. Trisomy 21 (Down syndrome) B. Marfan syndrome C. Oligohydramnios sequence (Potter syndrome) D. Fetal alcohol syndrome E. Prader-Willi syndrome

Correct The child described has facial features characteristic of fetal alcohol syndrome. Fetal alcohol spectrum disorders (FASD) are caused by the effects of maternal alcohol consumption during pregnancy. Fetal alcohol syndrome is the most clinically recognized form of FASD and is characterized by a pattern of minor facial anomalies, including a thin upper lip, a smooth philtrum, and a flat nasal bridge; other physical anomalies, such as clinodactyly; prenatal and postnatal growth retardation; and functional or structural central nervous system abnormalities. Children with Down syndrome have hypotonia, a flat face, upward and slanted palpebral fissures and epicanthic folds, and speckled irises (Brushfield spots); varying degrees of mental and growth retardation; dysplasia of the pelvis; cardiac malformations; a simian crease; short, broad hands; hypoplasia of the middle phalanx of the 5th finger; and a high, arched palate. Marfan syndrome is characterized by pectus carinatum or pectus excavatum, an arm span to height ratio >1.05, a positive wrist and thumb sign, limited elbow extension, pes planus, and aortic ascendens dilatation with or without aortic regurgitation. The bilateral renal agenesis seen with Potter syndrome leads to death shortly after birth. Other anomalies include widely separated eyes with epicanthic folds, low-set ears, a broad and flat nose, a receding chin, and limb anomalies. Finally, Prader-Willi syndrome is characterized by severe hypotonia at birth, obesity, short stature (responsive to growth hormone), small hands and feet, hypogonadism, and mental retardation.

A 62-year-old African-American male is admitted to the hospital for the third time in 6 months with heart failure. He has dyspnea with minimal activity. Echocardiography reveals an ejection fraction of 40%. Which one of the following combinations of medications is most appropriate for long-term management of this patient? (check one) A. Enalapril (Vasotec) plus digoxin B. Hydralazine plus isosorbide dinitrate C. Losartan (Cozaar) plus amlodipine (Norvasc) D. Spironolactone (Aldactone) plus bisoprolol (Zebeta)

Correct The combination of the vasodilators hydralazine and isosorbide dinitrate has been shown to be effective in the treatment of heart failure when standard treatment with diuretics, β-blockers, and an ACE inhibitor (or ARB) is insufficient to control symptoms or cannot be tolerated. This combination is particularly effective in African-Americans with NYHA class III or IV heart failure, with advantages including reduced mortality rates and improvement in quality-of-life measures. Digoxin, a long-time standard for the treatment of heart failure, is useful in reducing the symptoms of heart failure but has not been shown to improve survival. Amlodipine and other calcium channel blockers do not have a direct role in the treatment of heart failure.

A 17-year-old female presents to your office with anterior knee pain. She tells you she recently started a running program. She says the pain is worse running down hills, and is vaguely localized just medial to the patella. Examination of the knee shows no effusion or instability, and there is no joint-line pain or patellar tenderness. McMurray's maneuver is negative. Plain radiographs of the knee appear normal. Which one of the following would be most appropriate at this point? (check one) A. MRI of the knee B. Modification of her running program and a quadriceps and hip strengthening program C. Static stretching of the quadriceps and hamstrings prior to running D. A corticosteroid injection in the area of the pes anserine bursa

Correct This patient is suffering from patellofemoral pain syndrome, which causes anterior knee pain that is worse with running downhill. The examination is often normal, although there may be apprehension when the knee is extended with pressure over the patella and the patella will sometimes track laterally. Patellofemoral pain syndrome can be treated with exercises to strengthen the quadriceps and hips, and by using a knee sleeve with a doughnut-type cushion that the patella fits into. Static stretching would not address the problem. MRI would be indicated if there were joint-line pain or an unstable knee. Pes anserine bursitis usually causes pain and tenderness medially, below the joint line.

In 2009, the Centers for Medicare & Medicaid Services introduced the concept of Accountable Care Organizations primarily to accomplish which one of the following objectives? (check one) A. Prevent rates of reimbursement from growing faster than GDP B. Increase reimbursement to medically disadvantaged regions of the United States C. Require uninsured individuals to purchase health insurance at fair-market prices D. Deliver more efficient, high-quality services by encouraging cooperation between health care providers E. Reduce the power of integrated health systems to behave as monopolie

Correct The concept of Accountable Care Organizations (ACOs) was introduced in 2009 by the Centers for Medicare and Medicaid Services (CMS) to encourage doctors, hospitals, and other health care providers to work together to deliver high-quality care and spend health care dollars more wisely. The ACO concept, together with a shared savings program, has had difficulty penetrating smaller practices and more rural regions of the country. There is also concern that ACOs may allow larger systems to work as a monopoly as an unintended consequence. For this reason the Department of Justice and the Federal Trade Commission are monitoring these organizations as they develop. This new strategy of shared savings through coordinated health care is an alternative to the Sustainable Growth Rate (SGR) formula that CMS had previously hoped would contain health-care costs. The SGR was created to prevent Medicare rates from growing faster than the GDP. The high-profile topic of requiring individuals to carry health insurance has also been part of governmental reform initiatives but is not directly related to ACOs.

You have diagnosed chronic fatigue syndrome in a 32-year-old female. Her PHQ-9 is negative for depression. An evaluation for sleep disturbance and other comorbid disorders is also negative. Which one of the following would be the most effective treatment? (check one) A. Cognitive-behavioral therapy B. Interpersonal therapy C. Citalopram (Celexa) D. Methylphenidate (Ritalin)

Correct The criteria for chronic fatigue syndrome include fatigue for 6 months and a minimum of four of the following physical symptoms: impaired memory, postexertional malaise, muscle pain, polyarthralgia, tender lymph nodes, sore throat, new headaches, and unrefreshing sleep. Both cognitive-behavioral therapy and graded exercise therapy have been shown to improve fatigue levels, anxiety, work/social adjustment, and postexertional malaise (SOR A). Treatments that have not been shown to be effective include methylphenidate, melatonin, and galantamine. Citalopram has not been shown to be effective in the absence of a comorbid diagnosis of depression.

What is the specific antidote used to treat methanol poisoning? (check one) A. Ethanol B. Haloperidol C. Lorazepam (Ativan) D. Naloxone E. Thiamine

Correct The current management of methanol intoxication, depending on its severity, includes ethanol administration to inhibit the metabolism of methanol, hemodialysis to remove alcohol and its toxins, and vigorous management of metabolic acidosis with bicarbonate therapy. Ethanol is a competitive inhibitor of toxin metabolism and slows the formation of toxic metabolites, formaldehyde, and formic acid from methanol, permitting these products to be disposed of by ordinary metabolic or excretory pathways. It has a similar effect in ethylene glycol poisoning, slowing the formation of glycoaldehyde and glycolic, glyoxylic, and oxalic acids.

A 13-year-old female is brought to your office for evaluation of school difficulties and depressed mood. Her mother and older sister have both been diagnosed with depression. After a thorough history and physical examination, you diagnose major depressive disorder. You arrange for the patient to receive cognitive-behavioral therapy, but after 6 weeks her condition is only minimally improved.Which one of the following medications would be appropriate to add to this patient's treatment plan at this point? (check one) A. Fluoxetine (Prozac) B. Imipramine (Tofranil) C. Lithium D. Venlafaxine

Correct The diagnostic criteria for depression are the same for children and adults, although the manner in which these symptoms present may be different. Adolescents with depression are more likely to experience anhedonia, boredom, hopelessness, hypersomnia, weight change, alcohol or drug use, and suicide attempts. Psychotherapy should always be included as part of a treatment plan for depression in adolescents. Cognitive-behavioral therapy and interpersonal therapy are two modalities that have been proven effective in the treatment of adolescent depression. Medications should be considered for more severe depression or depression failing to respond to psychotherapy. A Cochrane review found that fluoxetine was the only agent with consistent evidence of effectiveness in decreasing depressive symptoms in adolescents. Consensus guidelines recommend fluoxetine, citalopram, or sertraline as first-line treatments for moderate to severe depression in children and adolescents. Escitalopram is also licensed for the treatment of depression in adolescents age 12 or older. All antidepressants have a boxed warning regarding an increased risk of suicide; therefore, close monitoring is recommended to assess for suicidality and other adverse effects, such as gastrointestinal effects, nervousness, headache, and restlessness. Tricyclic antidepressants were previously used to treat depression in children, but studies have shown little to no benefit in adolescents and children.

A 32-year-old gravida 2 para 1 with long-standing untreated hypertension presents at 8 weeks gestation for prenatal care. Her physical examination is normal except for a blood pressure of 156/114 mm Hg. Which one of the following would be most appropriate as initial treatment? (check one) A. Labetalol (Trandate) B. Lisinopril (Prinivil, Zestril) C. Losartan (Cozaar) D. Metoprolol (Lopressor, Toprol-XL) E. Nifedipine, immediate release (Procardia)

Correct The drug most often recommended as first-line therapy for hypertension in pregnancy is labetalol. Reports of an association of metoprolol with fetal growth restriction have given rise to the recommendation to avoid its use in pregnancy. Both ACE inhibitors and angiotensin-receptor blockers are contraindicated in pregnancy because of the risk of birth defects and fetal or neonatal renal failure. Immediate-release nifedipine is not recommended due to the risk of hypotension.

A 42-year-old female is troubled by her lack of interest in sex. She is generally healthy, takes no medications, and has regular menstrual periods. She is content with the emotional intimacy of her marriage and has had satisfying sexual interactions in the past. She does not have any religious or cultural barriers regarding her sexuality, and asks for ideas on how to improve her situation. Which one of the following has consistent evidence of benefit in cases such as this? (check one) A. Cognitive-behavioral therapy B. Viewing pornography C. Oral estrogen D. Oral sildenafil (Viagra) E. Topical testosterone

Correct This patient meets the criteria for hypoactive sexual desire disorder (HSDD). The incidence of this condition is variable based on the age, life stage, and culture of the patient, but is estimated to be present in about 5%-15% of the adult female population. This diagnosis includes two components: (1) recurrent deficiency or absence of sexual desire or receptivity to sexual activity, and (2) distress about such a deficiency. In menstruating women, oral estrogen and oral sildenafil have not been shown to be superior to placebo. Cognitive-behavioral therapy has been shown to be helpful for other sexual dysfunctions, but not with HSDD. Topical testosterone, in either patch or gel form, has shown consistent improvements in arousal, desire, fantasy, orgasm, and overall satisfaction in cases of HSDD.

An 85-year-old navy veteran presents to your office with a complaint of cough and dyspnea with exertion. He spent his entire career in ship maintenance and repair, and retired from the navy at the age of 45. His chest radiograph shows pleural plaques. He has a 20-pack-year smoking history, but quit at the age of 39. You suspect his problem is due to occupational exposure to which one of the following? (check one) A. Asbestos B. Beryllium C. Iron oxide D. Silica E. Uranium

Correct The inhalation of asbestos fibers may lead to a number of respiratory diseases, including lung cancer, asbestosis, pleural plaques, benign pleural effusion, and malignant mesothelioma. High-risk populations for asbestos exposure include individuals who worked in construction trades or as boilermakers, shipyard workers, or railroad workers, as well as U.S. Navy veterans. The occupational history helps to guide clinical suspicion in these high-risk populations. This patient is a retired U.S. Navy veteran who spent his entire career in ship maintenance and repair. The patient history is not consistent with berylliosis, silicosis, or uranium exposure. Berylliosis is an occupational disease related to mining and manufacturing. Silicosis is seen in sandblasters, miners, persons who have worked with abrasives, and several other occupations. Uranium exposure occurs after nuclear reactor leaks or blasts. Uranium compounds are also used in photography and as dyes or fixatures. The chemical toxicity involves nonmalignant damage to alveolar cells. Iron oxide exposure is not known to be related to lung disease.

Which one of the following intravenous agents is the best INITIAL management for hypercalcemic crisis? (check one) A. Furosemide B. Pamidronate C. Hydrocortisone D. Saline

Correct The initial management of hypercalcemic crisis involves volume repletion and hydration. The combination of inadequate fluid intake and the inability of hypercalcemic patients to conserve free water can lead to calcium levels >14-15 mg/dL. Because patients often have a fluid deficiency of 4-5 liters, delivering 1000 mL of normal saline during the first hour, followed by 250-300 mL/hour, may decrease the hypercalcemia to less than critical levels (<13 mg/dL). If the clinical status is not satisfactory after hydration alone, then renal excretion of calcium can be enhanced by saline diuresis using furosemide. Intravenous pamidronate, a bisphosphonate, reduces the hypercalcemia of malignancy and is best used in the semi-acute setting, since calcium levels do not start to fall for 24 hours. Glucocorticoids are useful in the treatment of hypercalcemia associated with certain malignancies (multiple myeloma, leukemia, several lymphomas, and breast cancer) or with vitamin D intoxication. The onset of action, however, takes several days, with the effect lasting days to weeks.

A 30-year-old female presents to your office with a clear nasal discharge, sneezing, nasal congestion, and nasal itching. She notes that these symptoms generally occur in the spring and fall. The most effective drug for treatment and prevention is? (check one) A. Cetirizine (Zyrtec) B. Cromolyn nasal spray (NasalCrom) C. Ipratropium nasal spray (Atrovent D. Montelukast (Singulair) E. Fluticasone nasal spray (Flonase)

Correct The initial treatment of mild to moderate allergic rhinitis should be an intranasal corticosteroid alone, with the use of second-line therapies for moderate to severe disease (SOR A). The adverse effects and higher cost of intranasal antihistamines, as well as their decreased effectiveness compared with intranasal corticosteroids, limit their use as first-or second-line therapy for allergic rhinitis. Moderate to severe disease not responsive to intranasal corticosteroids should be treated with second-line therapies, including antihistamines, decongestants, cromolyn, leukotriene receptor antagonists, and nonpharmacologic therapies such as nasal irrigation.

In which one of the following cardiac emergency cases should atropine be used? (check one) A. Symptomatic Mobitz type II atrioventricular block B. Cardiac arrest with pulseless electrical activity C. Asystolic cardiac arrest D. Acute cardiac ischemia and a heart rate <60 beats/min E. Sinus bradycardia with hypotension

Correct The main use of atropine in cases of cardiac arrest is for symptomatic bradycardia. It has little effect with complete heart block and Mobitz type II atrioventricular block. It is not recommended or effective for cardiac arrest with pulseless electrical activity or in cases of asystole. It has been removed from these algorithms by the ACLS committee. During an acute myocardial infarction or acute cardiac ischemia, an increase in heart rate may increase the amount of ischemia.

An 82-year-old white male has a cardiopulmonary arrest while mowing his lawn and his heart rhythm is restored after 8 minutes of CPR by a neighbor. He is now your patient in the coronary care unit. He is on a ventilator and has severe hypoxic encephalopathy. Echocardiography shows an ejection fraction of 12% as a result of the massive anterior myocardial infarction he sustained. Your neurology consultant confirms that the patient will never again be able to meaningfully communicate, and will be ventilator-dependent. Prior to this, the patient had been living independently and had no health problems. He has no living relatives, and his attorney confirms that he has no written advance directives. The neighbor, who is a close friend, tells you that on several occasions recently he and the patient had discussed such a scenario, and that the patient had said that if he had little chance of a meaningful recovery he would not want to remain on life support. In consultation with the hospital ethics committee, which one of the following would be most appropriate in this case? (check one) A. Transfer care of the patient to another physician B. Ask a court to appoint a guardian to make medical decisions C. Withdraw life support D. Defer the decision regarding life support to the hospital attorney E. Ask the patient's attorney to decide whether to terminate life support

Correct The most common form of advance directive is a patient's conversations with relatives and friends, and these carry the same ethical and legal weight as written directives. Neither the hospital attorney nor the patient's personal attorney, in the absence of a previous discussion with the patient, has the ethical or legal authority to make the decision. Since the patient has previously expressed his wishes, it is unnecessary to have the court appoint a surrogate decision maker. Care should be transferred to another physician only if the original physician has a philosophical or religious objection to carrying out the patient's wishes.

A 9-year-old male is brought to your office because he has developed a limp and refuses to bear weight on his right leg. On examination he has a temperature of 38.6°C (101.5°F) and pain with range of motion of the right hip. His WBC count and erythrocyte sedimentation rate are both elevated. A radiograph of the right hip is normal. Which one of the following would be most appropriate at this point? (check one) A. A repeat radiograph in 48 hours B. Ultrasonography of the right hip C. CT of the right hip D. MRI of the right hip E. A bone scan of the lumbar spine, right hip, and right femur

Correct The most likely diagnosis is septic arthritis of the hip. Ultrasonography is highly sensitive for the effusion seen in septic arthritis, which can be aspirated to confirm the diagnosis (SOR A). It is important to diagnose this problem as soon as possible. Clinical features of septic arthritis include an oral temperature >38.5°C (101.3°F), refusal to bear weight on the affected leg, an erythrocyte sedimentation rate >40 mm/hr, a peripheral WBC count >12,000/mm3, and a C-reactive protein level >20 mg/L. If ultrasonography is negative, a bone scan should be done. CT of the hip is indicated to visualize cortical bone. MRI is especially valuable for osteomyelitis.

A 44-year-old male with papulopustular rosacea sees you for follow-up. You have been treating his condition with topical azelaic acid (Finacea), and although his condition is improved he is not satisfied with the results. You suggest adding which one of the following oral medications? (check one) A. Clarithromycin (Biaxin) B. Clindamycin (Cleocin) C. Doxycycline D. Erythromycin E. Metronidazole (Flagyl

Correct The only FDA-approved oral treatment for acne rosacea is doxycycline at a subantimicrobial dosage (40 mg daily). This does not contribute to antibiotic resistance, even when used over several months, and is better tolerated than higher dosages. Other antibiotics have limited and low-quality supporting evidence of efficacy and may lead to antibiotic resistance.

During a well child examination, you notice that a 2-month-old male has a flattened left occiput. His records show that his skull was normally shaped at birth. Further evaluation shows that the left frontal region is more prominent than the right, and the left ear is slightly forward of its expected position. The infant seems comfortable rotating his head to either side while being held in his mother's arms. Which one of the following would be appropriate at this time? (check one) A. Recommend that the infant sleep in a prone position, and follow up in 1 month B. Educate the parents about positioning and follow up in 2 months C. Order physical therapy D. Order CT of the head E E. Refer for surgical evaluation

Correct The parallelogram shape of this infant's head is typical of positional skull deformity, also known as benign positional molding or occipital plagiocephaly. This condition has been estimated to be present in at least 1 in 300 infants, with some studies showing milder variants in up to 48% of healthy infants. The incidence of positional skull deformity is increased in children who sleep in the supine position, but switching to prone sleeping is not recommended because this would increase the risk of sudden infant death syndrome. The deformity can be prevented by routine switching of the dependent side of the infant's head. Supervised "tummy time" for 30-60 minutes each day can also decrease the amount of flattening and can increase the child's motor development. Children who have positional skull deformity should also be screened for torticollis. This condition can prevent correct positioning and is remedied with physical therapy techniques. Positional skull deformity should be differentiated from cranial synostosis, which is the result of abnormal fusion of one or more of the sutures between the skull bones. Ipsilateral frontal bossing and ear advancement are not seen, resulting in a trapezoid-shaped head. Most infants with positional skull deformity improve within 2-3 months with the institution of positional changes and tummy time. If the condition does not significantly improve after this amount of time, referral to a pediatric neurosurgeon with expertise in craniofacial malformations would be appropriate.

A 15-year-old male presents to the emergency department at 10 p.m. with a 2-hour history of severe, acute scrotal pain associated with vomiting. On examination the right testicle is swollen. Ultrasonography is inconclusive. Which one of the following would be most appropriate at this point? (check one) A. Repeat ultrasonography in the morning B. Antibiotics C. Corticosteroids D. Scrotal support E. Immediate surgical consultation

Correct The patient has typical signs and symptoms of testicular torsion despite inconclusive ultrasonography. Surgical exploration is necessary because the testicle can be salvaged if the torsion is repaired within 6hours of symptom development (SOR C).

One year after being diagnosed with early Alzheimer's disease, one of your long-time patients develops symptomatic carotid stenosis. A vascular surgeon has recommended surgical treatment, but the patient's family is uncertain whether he should have the surgery and if he is capable of making the decision. The children are evenly split in their opinion regarding the surgery, and they ask for your input. Which one of the following is true regarding this situation? (check one) A. The patient is incapable of making this decision because of his dementia B. The Mini-Mental State Examination score determines competence C. The patient should be evaluated by a psychiatrist D. A judicial determination of competence should be obtained E. The patient's decision-making capacity can be adequately assessed by clinical evaluation

Correct The primary care physician can assess decision-making capacity based on the patient's ability to reason, communicate, understand the proposed treatment, and grasp the consequences of accepting or declining the suggested treatment. Formal mental status testing and determination of capacity are different functions. Accurate mental status testing is helpful for assessing the capacity to make decisions, but there is not a specific score that determines capacity. However, there is a certain level of cognitive impairment where a patient simply lacks any ability to receive and process health information. At somewhat higher levels of cognition a patient might lack specific mental abilities, but still be able to satisfy the requirements for making treatment decisions. A recent meta-analysis showed that Mini-Mental State Examination (MMSE) scores below 20 increase the likelihood of incapacity (LR 6.3), scores of 20-24 have no effect (LR 0.87), and scores greater than 24 significantly lower the likelihood of incapacity (LR 0.17).

The primary indication for joint replacement surgery in patients with osteoarthritis is? (check one) A. intractable pain B. joint laxity C. limited range of motion D. recurrent subluxation

Correct The primary indication for joint replacement surgery in patients with osteoarthritis is intractable pain, which is almost always relieved by the surgery. Joint replacement may also be appropriate for patients with significant limitations of joint function or with altered limb alignment. Range of motion, joint laxity, and recurrent subluxation relate to musculotendinous function, and are not reliably improved by joint replacement.

A 35-year-old primigravid schoolteacher awakens with a rash clinically consistent with varicella early in the 38th week of her pregnancy. She had a negative varicella titer early in her pregnancy. The clinical course is mild and all vesicles have either crusted over or healed 1 week later. She has an uncomplicated labor and vaginal delivery at 40 weeks gestation, and delivers a healthy-appearing male. Of the following options, which one is the most appropriate initial management for the newborn? (check one) A. Intravenously administered varicella immune globulin B. A weight-appropriate dose of intravenous acyclovir (Zovirax) C. Varicella vaccine D. Combination treatment with varicella vaccine, intravenous acyclovir, and varicella immune globulin E. Close observation only

Correct The result of neonatal varicella infection can be catastrophic, with a fatality rate approaching 30%. Maternal immunity is ideal, but since varicella vaccination is contraindicated during pregnancy the best alternative is advising the patient to avoid contact with infected individuals until safe postpartum immunization is possible. Maternal varicella infection is particularly problematic during weeks 13-20 of pregnancy (resulting in a 2% risk of congenital varicella in the newborn) and when the onset of maternal symptoms occurs from 5 days before until 2 days after delivery. Administration of varicella immune globulin to the expectant mother has not been shown to benefit the fetus or infant, but because pregnancy can increase the risk of serious complications in the mother the Advisory Committee on Immunization Practices (ACIP) recommends that administration to pregnant women be considered following known exposure. The ACIP also recommends that term infants born within the 7-day window described above, as well as all preterm infants, receive varicella immune globulin, and that those who develop any signs of varicella infection also be given intravenous acyclovir. Term infants delivered more than 5 days after the onset of maternal varicella are thought to have adequate passive immunity for protection and the expected benign course generally requires only observation.

In an adult who has a critical illness but no history of cardiac disease, the threshold for transfusion of red blood cells should be a hemoglobin level of ? (check one) A. 6 g/dL B. 7 g/dL C. 8 g/dL D. 9 g/dL E. 10 g/dL

Correct The threshold for transfusion of red blood cells should be a hemoglobin level of 7 g/dL in adults and most children.

Which class of medication is first-line therapy for uncomplicated depression during pregnancy? (check one) A. Monoamine oxidase inhibitors (MAOIs) B. SSRIs C. SNRIs D. Stimulants E. Tricyclic antidepressants

Correct The treatment of depression in pregnancy is determined by the severity of the symptoms and any past history of treatment response. For women who have a new onset of mild or moderate depression, it may be best to start with nonpharmacologic treatments such as supportive psychotherapy or cognitive-behavioral therapy. These interventions may improve the depression enough that the patient will not need medications. However, in situations where pharmacologic treatment is clearly indicated, SSRIs are thought to have the best safety profile. Fluoxetine, sertraline, and citalopram have extensive data to support their safety in pregnancy and should be considered first line. Paroxetine is the one SSRI that is thought to carry an increased risk of congenital malformations with first-trimester exposure and should be avoided. Tricyclic antidepressants are class D in pregnancy. SNRIs do not have as much safety data as SSRIs to support their use in pregnancy and would be considered a second-line choice. MAOIs are known teratogens and should be avoided in pregnancy. Stimulants are not first-line agents and should be avoided in pregnancy.

For several years, a hypertensive 65-year-old female has been treated with hydrochlorothiazide, 25 mg/day; atenolol (Tenormin), 100 mg/day; and hydralazine, 50 mg 4 times/day. Her blood pressure has been well controlled on this regimen. Over the past 2 months she has experienced malaise, along with diffuse joint pains that involve symmetric sites in the fingers, hands, elbows, and knees. A pleural friction rub is noted on examination. Laboratory testing shows that the patient has mild anemia and leukopenia, with a negative rheumatoid factor and a positive antinuclear antibody (ANA) titer of 1:640. Which one of the following would be the most appropriate INITIAL step? (check one) A. Replace hydrochlorothiazide with furosemide (Lasix) B. Discontinue hydralazine C. Start prednisone, 40 mg/day orally D. Start hydroxychloroquine (Plaquenil), 400 mg/day E. Order renal function studies and anticipate that a renal biopsy will be needed

Correct There are many drugs that can induce a syndrome resembling systemic lupus erythematosus, but the most common offenders are antiarrhythmics such as procainamide. Hydralazine is also a common cause. There is a genetic predisposition for this drug-induced lupus, determined by drug acetylation rates. Polyarthritis and pleuropericarditis occur in half of patients, but CNS or renal involvement is rare. While all patients with this condition have positive antinuclear antibody titers and most have antibodies to histones, antibodies to double-stranded DNA and decreased complement levels are rare, which distinguishes drug-induced lupus from idiopathic lupus. The best initial management for drug-induced lupus is to withdraw the drug, and most patients will improve in a few weeks. For those with severe symptoms, a short course of corticosteroids is indicated. Once the offending drug is discontinued, symptoms seldom last beyond 6 months.

Which one of the following has good evidence of effectively improving borderline personality disorder? (check one) A. SSRIs B. Second-generation antipsychotics C. Omega-3 fatty acids D. No currently available pharmacotherapy

Correct There are no proven therapies to reduce the severity of borderline personality disorder (SOR A). The most promising psychological therapy is dialectic behavioral therapy (DBT). DBT is a multi-faceted program specifically designed to treat borderline personality disorder. The few, small studies of DBT found improvement in many symptoms of borderline personality disorder, but long-term data is lacking. Another promising therapy is psychoanalytic-oriented day hospital therapy. Again, study sizes have been small and data cannot be extrapolated to the population as a whole.Omega-3 fatty acids, second-generation antipsychotics, and mood stabilizers have been shown to be helpful for some symptoms of borderline personality disorder but not for overall severity. Their benefits are based on single-study results and side effects were not addressed in the studies. SSRIs are not recommended for borderline personality disorder unless there is a concomitant mood disorder.

An 87-year-old female is brought to the emergency department after losing consciousness at the dinner table. Her history indicates recent unintentional weight loss. Further evaluation ultimately reveals a large mass at the head of the pancreas and extensive metastasis to numerous organs, including the brain. Her life expectancy is estimated to be 2-3 weeks. The patient chooses to receive hospice care but becomes very depressed. Which one of the following would be best for improving her depression? (check one) A. Electroconvulsive therapy B. Methylphenidate (Ritalin) C. Mirtazapine (Remeron) D. Fluoxetine (Prozac) E. Nortriptyline (Pamelor)

Correct There is good evidence that psychostimulants reduce symptoms of depression within days, making methylphenidate a good choice for this patient (SOR B). Electroconvulsive therapy is contraindicated due to her brain lesions. Mirtazapine, fluoxetine, and nortriptyline all take at least 3-4 weeks to have any antidepressant effects, and would not be appropriate given the patient's life expectancy (SOR B).

A 20-year-old female distance runner presents with a 1-month history of left knee pain. The pain is worse with her first few steps in the morning and when going down stairs. Examination of the knee reveals no deformity, effusion, or ligamentous laxity. The knee joint and surrounding tissues are not tender to palpation, with the exception of an area 2 cm proximal to the left lateral joint line. What is the most likely cause of this patient's pain? (check one) A. Osteoarthritis of the knee joint B. Pes anserine bursitis C. Iliotibial band syndrome D. Chronic lateral meniscal tear E. Osgood-Schlatter disease

Correct This case illustrates the classic history and physical findings of iliotibial band syndrome. Pain occurs most frequently at the site where the tendon crosses over the lateral femoral epicondyle. With osteoarthritis or a meniscal tear there would be pain in the joint space with palpation. Osgood-Schlatter disease is more common in younger adolescents and is characterized by tenderness of the tibial tubercle at the distal insertion of the patellar ligament. Pes anserine bursitis is characterized by pain in the medial knee distal to the joint space, at the conjoined tendon of the sartorius, gracilis, and semitendinosus.

A 67-year-old female hospitalized with pneumonia develops the rapid onset of dyspnea, pleuritic chest pain, tachypnea, and hypoxemia not responding to oxygen and requiring intubation. A physical examination is notable for rales throughout both lung fields with no peripheral edema noted. A chest radiograph shows bilateral pulmonary infiltrates. Her BNP level is 90 ng/L. Which one of the following is the most likely reason for her worsening clinical situation? (check one) A. Heart failure B. Hypersensitivity pneumonitis C. Acute respiratory distress syndrome D. Pulmonary embolus E. Pneumothorax

Correct This patient demonstrates classic findings for acute respiratory distress syndrome (ARDS). In many cases ARDS must be differentiated from heart failure. Heart failure is characterized by fluid overload (edema), jugular venous distention, a third heart sound, an elevated BNP level, and a salutary response to diuretics. A BNP level <100 pg/mL can help rule out heart failure (SOR A). In addition, a patient with ARDS would not have signs of left atrial hypertension and overt volume overload. Hypersensitivity pneumonitis is usually preceded by exposure to an inciting organic antigen such as bird feathers, mold, or dust. Pulmonary embolus, while certainly in the differential, is unlikely to cause such dramatic radiographic findings. Pneumothorax would be seen on the chest radiograph.

A 40-year-old male presents with a sudden onset of unilateral peripheral facial nerve weakness 1 hour ago. Which one of the following is most likely to shorten his symptoms? (check one) A. Corticosteroid therapy B. Antiviral therapy C. Thrombolytic therapy D. Hyperbaric oxygen therapy E. Facial nerve decompression

Correct This patient has Bell's palsy. Only corticosteroids have been shown to improve the outcome. Antiviral agents have little value in the treatment of Bell's palsy. Thrombolytic therapy may be useful for a patient with central facial nerve weakness if it is due to a vascular event (level of evidence 3; SOR A).

An 18-year-old basketball player comes to your office for evaluation of finger pain. During a basketball game yesterday he was hit on the tip of his right second digit and now has finger pain and difficulty moving his finger. On examination he has bruising and tenderness over the distal interphalangeal (DIP) joint. His DIP joint is in the flexed position and he is unable to extend the joint. A radiograph shows a fracture at the dorsal surface of the proximal distal phalanx involving 10% of the joint space. What is the most appropriate management of this injury? (check one) A. Taping the finger to the adjacent finger B. Splinting in full extension C. Splinting in 45° of flexion D. Urgent surgical management E. Intermittent splinting for comfort

Correct This patient has a mallet fracture. These fractures are caused by an axial load to the tip of an extended finger that causes forced flexion at the distal interphalangeal (DIP) joint. This leads to a fracture at the dorsal surface of the proximal distal phalanx where the terminal finger extensor mechanism inserts. The most appropriate treatment of a mallet fracture is to splint the DIP joint in extension for 8 weeks. The joint should remain in full extension for optimal healing. Any flexion of the finger may affect healing and extend the treatment time. Surgical management has been recommended for fractures that involve more than 30% of the joint space, but a small study showed there was no difference in outcomes compared to treatment with extension splints. Buddy taping would not offer enough support to maintain the finger in extension at all times.

A 32-year-old female has a 3-week history of depressed mood. She reports markedly diminished interest or pleasure in most activities, fatigue, a diminished ability to concentrate, and insomnia. She has had recurrent suicidal thoughts, but has no specific plan. Further investigation reveals a past history of several hypomanic episodes lasting 4-5 days, characterized by a persistently elevated, expansive mood. During these episodes she needed little sleep, was talkative, met multiple goals, and had trouble keeping up with the thoughts that were running through her head. She was treated with lithium in her early twenties but she stopped taking it because it stifled her artistic creativity. She currently takes no medication. Her physical examination is unremarkable. Results from comprehensive laboratory studies, including a urine toxicology screen, are also normal. Which one of the following is most appropriate for her current depressive symptoms? (check one) A. Aripiprazole (Abilify) B. Venlafaxine C. Divalproex (Depakote) D. Divalproex and bupropion (Wellbutrin) E. Lithium and paroxetine (Paxil)

Correct This patient has bipolar II disorder. She has a history of hypomanic episodes as well as major depression, with no history of a manic or mixed episode. Among the pharmacologic options listed, only divalproex and lithium are indicated for treating bipolar depression or acute mania, and for maintenance. They should be given as single agents, however, not in combination with other drugs. No evidence supports combination therapy or the addition of an antidepressant in the acute phase of depression. In a study of patients with bipolar II disorder, initially adding paroxetine or bupropion to the mood stabilizer was no more effective than using lithium or valproate. An SSRI or bupropion can be added if a therapeutic dosage of a mood stabilizer does not resolve symptoms and the patient is not in a mixed state. Tricyclic antidepressants and antidepressants with dual properties, such as venlafaxine, should be avoided because they may induce mania. Aripiprazole is indicated for acute mania but not for bipolar depression.

A 54-year-old female concert pianist presents to your office with a 9-month history of searing pain and bilateral paresthesias in the distribution of her median nerve. She says that the pain frequently radiates as far as her shoulder, and that her fingers feel swollen even though they look normal. She states that she has worsening paresthesias at night and often finds herself flicking her wrist in an attempt to alleviate her symptoms. The patient's symptoms are reproducible with wrist flexion and she exhibits mild weakness of the abductor pollicis brevis on examination. She has been wearing neutral wrist splints at night for the last 8 weeks and has also been taking oral NSAIDs, resulting in only minimal relief. She is in the middle of her concert season and is unable to take time off for a surgical procedure. Which one of the following therapies will provide this patient with the longest symptom relief? (check one) A. Full rest for 8 weeks B. Full-time cock-up slinging for 8 weeks C. Physical therapy D. Oral corticosteroids E. Local corticosteoid injection

Correct This patient has carpal tunnel syndrome. Initial conservative approaches for mild to moderate symptom relief include full-time splinting for 8 weeks (SOR B) and oral corticosteroids. However, studies suggest that local corticosteroid injections offer symptom relief for 1 month longer than oral corticosteroid therapy and some individuals experience relief for up to 1 year. Severe or chronic symptoms usually require surgical intervention for nerve decompression. Physical therapy is not recommended, and full rest is unlikely in a person in a high-risk occupation for overuse syndromes.

A 48-year-old male presents with a 4-week history of rectal pain associated with minimal rectal bleeding. On examination there is a small tear of the anorectal mucosa at the 6 o'clock position. The most appropriate initial treatment would be topical? (check one) A. Botulinum toxin B. Clobetasol (Temovate) C. Capsaicin (Capzasin-HP, Zostrix) D. Nitroglycerin

Correct This patient has classic findings for acute rectal fissure. Although patients often require an internal sphincterotomy, nonsurgical measures that relax the sphincter have proven helpful. Botulinum toxin injected into the internal sphincter has proven most beneficial, but topical preparations are not yet available and have not been shown to be effective for this problem. Corticosteroid creams may decrease the pain temporarily, but potent fluorinated corticosteroid creams such as clobetasol are not indicated in the treatment of fissure. Capsaicin cream can be helpful for pruritus ani, but not for anal fissures. Drugs that dilate the internal sphincter, including diltiazem, nifedipine, and nitroglycerin ointment, have proven to be beneficial in healing acute fissures, but usually have to be compounded by a pharmacist.

A 41-year-old male presents with a complaint of headaches for the past 6 weeks. He has severe, sharp, right-sided periorbital pain 3-4 days each week. When these headaches occur his right eye gets watery, his right nostril feels clogged, and his forehead feels sweaty. When he gets the headaches he takes four 200-mg ibuprofen tablets and goes into a dark, quiet room. The headaches usually resolve in about 90 minutes. Currently he is feeling well and his examination is completely normal. What type of headache does he most likely have? (check one) A. Medication overuse headache B. Migraine C. Paroxysmal hemicrania D. Temporal arteritis E. Cluster headache

Correct This patient has cluster headaches. Most people with cluster headaches are male. These headaches typically present with severe unilateral pain that lasts from 15 minutes to 3 hours. The pain is generally extremely sharp, continuous, and incapacitating. In addition to the pain, the headaches are associated with at least one of the following ipsilateral signs: conjunctival injection, lacrimation, nasal congestion, miosis or ptosis, eye edema, and forehead and facial sweating. Patients may also have a sense of restlessness or agitation. The headaches occur anywhere from every other day up to 8 times a day, often in cycles for 4-12 weeks. Cluster headaches respond to most of the same medications as migraine headaches (DHE, ergotamines, triptans). They also respond well to 100% oxygen therapy. Paroxysmal hemicranias are very unusual and present with a similar type of pain, but the attacks are usually short and they are more common in women. Medication rebound headaches tend to be diffuse, bilateral, almost daily headaches. These occur in people who are overusing medications, and they tend to get worse with physical or mental exertion. Temporal arteritis usually occurs in older adults. Migraines are also often unilateral but they are usually pulsatile, and are associated with nausea and vomiting or photophobia and phonophobia.

A 36-year-old female presents with a several-week history of polyuria and intense thirst. She currently takes no medications. On examination her blood pressure and pulse rate are normal, and she is clinically euvolemic. Laboratory tests, including serum electrolyte levels, renal function tests, and plasma glucose, are all normal. A urinalysis is significant only for low specific gravity. Her 24-hour urine output is >5 L with low urine osmolality. The most likely cause of this patient's condition is a deficiency of (check one) A. angiotensin II B. aldosterone C. renin D. insulin E. arginine vasopressin

Correct This patient has diabetes insipidus, which is caused by a deficiency in the secretion or renal action of arginine vasopressin (AVP). AVP, also known as antidiuretic hormone, is produced in the posterior pituitary gland and the route of secretion is generally regulated by the osmolality of body fluid stores, including intravascular volume. Its chief action is the concentration of urine in the distal tubules of the kidney. Both low secretion of AVP from the pituitary and reduced antidiuretic action on the kidney can be primary or secondary, and the causes are numerous. Patients with diabetes insipidus present with profound urinary volume, increased frequency of urination, and thirst. The urine is very dilute, with an osmolality <300 mOsm/L. Further workup will help determine the specific type of diabetes insipidus and its cause, which is necessary for appropriate treatment. Low levels of aldosterone, plasma renin activity, or angiotensin would cause abnormal blood pressure, electrolyte levels, and/or renal function. Insulin deficiency results in diabetes mellitus.

A 50-year-old female reports vaginal dryness, burning, and pain with penetration during sexual intercourse. On examination she is noted to have pale, dry vaginal epithelium that is smooth and shiny with loss of most rugation. Which one of the following treatments is most likely to be effective for her sexual dysfunction? (check one) A. Cognitive-behavioral therapy B. Vaginal estrogen C. Testosterone therapy D. Bupropion (Wellbutrin) E. Sildenafil (Viagra)

Correct This patient has genitourinary syndrome of menopause (formerly termed vulvovaginal atrophy) based on her symptoms and examination. Estrogen therapy is highly effective for dyspareunia related to genitourinary syndrome of menopause, with the vaginal route preferred over systemic therapy if vaginal dryness is the primary concern. Bupropion and sildenafil may benefit women with sexual dysfunction induced by antidepressant medications. Data on the benefit of testosterone therapy is limited and inconsistent and lacks long-term information about safety. Cognitive-behavioral therapy has been shown to effectively treat low sexual desire, but does not affect the physiologic changes associated with genitourinary syndrome of menopause.

A 54-year-old female presents with painful sores in her mouth that appeared a few days ago. She has had some trouble eating due to the pain, but she is able to swallow without difficulty. She also began to have some pain around her right ear today. She has no fever, chills, nasal congestion, cough, or difficulty hearing. Her medical history is significant only for an anxiety disorder treated with sertraline (Zoloft). On examination her vital signs are all normal. You see vesicles on the right side of the hard palate and she has a swollen, red right pinna, with vesicles in the external auditory canal. The organism responsible for this condition is (check one) A. coxsackievirus B. Epstein-Barr virus C. group A Streptococcus D. herpes simplex virus E. varicella zoster virus

Correct This patient has herpes zoster oticus, which is also known as Ramsay Hunt syndrome when associated with a facial nerve palsy. It is caused by reactivation of the varicella-zoster virus (VZV) in the geniculate ganglion of the facial nerve. Typical symptoms include painful vesicles on one side of the palate and the ipsilateral ear. When the reactivation involves other branches of the facial nerve it can result in a unilateral facial herpetiform rash that may also involve the anterior two-thirds of the tongue, taste disturbance, and reduced lacrimation. If the nearby cochlear and vestibular nerves become involved, patients may also experience hearing loss, tinnitus, nausea, vomiting, and vertigo. The diagnosis is usually made clinically, but if confirmation is needed polymerase chain reaction testing of vesicular fluid or of a swab of the base of an ulcer may be done. Treatment includes antivirals (acyclovir, valacyclovir) and prednisone, and is more effective when started sooner in the course of illness. Herpes simplex virus (HSV) can cause oral vesicles and ulcers, but the distribution of vesicles in the ear and the mouth of this patient is not typical for HSV. Epstein-Barr virus can cause leukoplakia of the mouth but not vesicles and is typically associated with systemic signs of illness. Group A Streptococcus causes throat pain and fever, not vesicles. Coxsackievirus causes oral vesicles and ulcers but is usually associated with fever and does not typically involve the ear.

A 70-year-old male without underlying lung disease presents with a 36-hour history of fever, body aches, cough, and dyspnea. He did not receive influenza vaccine this year, and was recently exposed to his grandson who had influenza. On examination the patient has a temperature of 38.8°C (101.8°F), a blood pressure of 90/50 mm Hg, a heart rate of 110 beats/min, and an O2 saturation of 87% on room air. A nasal swab rapid antigen test is negative, and his WBC count is 15,000/mm3 (N 4300-10,800). A viral culture is sent to the laboratory. A chest radiograph shows a large lobar pneumonia. You hospitalize the patient and initiate? (check one) A. ceftriaxone (Rocephin) and azithromycin (Zithromax) B. levofloxacin (Levaquin) C. oseltamivir (Tamiflu) D. oseltamivir, ceftriaxone, and azithromycin E. oseltamivir, ceftriaxone, azithromycin, and vancomycin (Vancocin)

Correct This patient has pneumonia, sepsis, and suspected coinfection with influenza. Although the rapid antigen-based nasal swab was negative, false-negative rates may be as high as 70% and this test should not be relied upon to rule out influenza. Treatment should include both antiviral and antibacterial agents that include coverage against methicillin-resistant Staphylococcus aureus (MRSA), the most common bacterial pathogen isolated from critically ill patients with coinfection. Oseltamivir, ceftriaxone, azithromycin, and vancomycin should be initiated empirically for the pneumonia and sepsis. The criteria for sepsis are satisfied by a temperature >38.3°C, a WBC count >12,000/mm3, a respiratory rate >20/min, and a source of probable infection.

An 80-year-old male nonsmoker with Parkinson's disease is treated for community-acquired pneumonia with azithromycin (Zithromax), 500 mg/day for 10 days. On follow-up the patient feels better but still has a productive cough. A repeat chest radiograph reveals a single thin-walled cavity lesion in the left lower lobe. It would be most appropriate to replace this patient's azithromycin with? (check one) A. Doxycycline B. Clindamycin (Cleocin) C. Metronidazole (Flagyl) D. Trimethoprim/sulfamethoxazole (Bactrim, Septra)

Correct This patient most likely has an anaerobic bacterial infection. Penicillin was used to treat these infections in the past, but because of the emergence of β-lactamase-producing organisms, clindamycin is now the drug of choice. Clindamycin has broader coverage against both pulmonary anaerobes and facultative aerobes such as Staphylococcus aureus and Klebsiella, which are often seen with lung abscesses. Metronidazole has anaerobic coverage, but not for the anaerobic species often involved in pulmonary infections, and is therefore associated with a high failure rate when used to treat lung abscesses. Doxycycline does not cover anaerobes. Trimethoprim/sulfamethoxazole is also not considered a good anaerobic antibiotic.

A 62-year-old female with known systolic heart failure has a 2-month history of increased fatigue and worsening shortness of breath with ambulation. She says she has adhered to her medication regimen. Her oxygen saturation is 96% on room air and a physical examination is within normal limits. Laboratory studies, chest radiographs, and an EKG are ordered. The echocardiogram shows an ejection fraction of 35% and normal right heart function. Her estimated pulmonary pressure is 45 mm Hg. The best option for treatment of her pulmonary hypertension at this point is to (check one) A. add a vasodilator B. begin oxygen therapy C. recommend lifelong anticoagulation D. maximize treatment for heart failure E. schedule right heart catheterization

Correct This patient has pulmonary hypertension due to left heart failure. The recommended treatment is to maximize treatment for her heart failure and any other comorbidities. Vasodilators are not recommended in the treatment of pulmonary hypertension due to left heart failure and may be harmful (SOR C). Oxygen therapy is recommended only for patients with hypoxia (SOR C). Lifelong anticoagulation is recommended if pulmonary hypertension is due to chronic thromboembolic disease but not if it is due to left heart failure (SOR C). Anticoagulation is not recommended in systolic left heart failure unless there is another indication. Right heart catheterization is not recommended for pulmonary hypertension due to left heart disease because vasodilators are not a treatment option. Right heart catheterization is recommended in pulmonary hypertension prior to initiating vasodilator therapy in appropriate patients (SOR C).

A 45-year-old female with no significant past medical history presents to your office with 2 weeks of worsening pain in her right arm. For the past 2 months she has worked on a plastics manufacturing assembly line. A physical examination reveals no swelling and a normal range of motion. She has normal strength in the upper extremity but she experiences increased pain with extension of her right wrist against resistance. Palpation reveals marked tenderness over the lateral epicondyle of the right arm. Which one of the following is most likely to improve the patient's long-term outcome? (check one) A. Physical therapy B. Regular physical activity using her hands and arms C. Use of an inelastic, nonarticular proximal forearm strap D. Modifying her work routines

Correct This patient has signs and symptoms of lateral epicondylitis, also known as tennis elbow, or alternatively as lateral epicondylalgia to reflect the noninflammatory nature of the condition. This is an overuse tendinopathy of the common extensor tendon origin of the lateral elbow. Conservative care that includes offloading the involved tendons is the key to improving outcomes at 1 year, which would mean modifying this patient's work. Physical therapy can improve pain and function in the short term, but has not been shown to improve long-term outcomes at 1 year in randomized trials (SOR A). The evidence is weaker for bracing, with some studies showing improved pain and function at 3-6 weeks (SOR B). Recent randomized, controlled trials have made it clear that while corticosteroid injections reduce acute pain for up to 6 weeks, their use increases rates of poor long-term outcomes (SOR A).

A patient who has terminal metastatic lung cancer with bony metastases is being cared for at home and using hospice services. The hospice nurse calls you during the night because the family had called her to come to the house. When she arrived she found the patient acutely agitated, confused, and disoriented, and he does not recognize his family members. The patient is trying to hit his caretakers, who are distressed by the situation. In addition to checking for underlying causes of these acute symptoms, which one of the following is most appropriate for managing this problem? (check one) A. Amitriptyline B. Haloperidol C. Scopolamine D. Trazodone (Oleptro)

Correct This patient is experiencing delirium, which is common in the last weeks of life, occurring in 26%-44% of persons hospitalized with advanced cancer and in up to 88% of persons with a terminal illness. In studies of a palliative care population it was possible to determine a cause for delirium in less than 50% of cases. There is a consensus based on observational evidence and experience that antipsychotic agents such as haloperidol are effective for the management of delirium, and they are widely used. However, there have been few randomized, controlled trials to assess their effectiveness. While benzodiazepines are used extensively in persons with delirium who are terminally ill, there is no evidence from well-conducted trials that they are beneficial. Trazodone is an antidepressant that is sometimes used for insomnia. Scopolamine is an anticholinergic that is used to reduce respiratory secretions in hospice patients, but its anticholinergic side effects would increase delirium severity. Amitriptyline also has significant anticholinergic properties.

A 25-year-old male has developed a painless ulcer on the glans of his penis. After an appropriate examination and testing you diagnose primary syphilis and treat him with 2.4 million units of benzathine penicillin intramuscularly in a single dose. Eight hours later, while you are working the evening clinic, he returns because he has a fever of 100.6°F and a bad headache, which he rarely gets. He says he "aches all over." Which one of the following would be most appropriate at this time? (check one) A. Three blood cultures from different sites at 30-minute intervals B. CT of the head C. A lumbar puncture D. Doxycycline, 100 mg orally twice a day for 14 days E. Reassurance and antipyretics

Correct This patient is experiencing the Jarisch-Herxheimer reaction—an acute, transient, febrile reaction that occurs within the first few hours after treatment for syphilis. The condition peaks at 6-8 hours and disappears within 12-24 hours after therapy. The temperature elevation is usually low grade, and there is often associated myalgia, headache, and malaise. It is usually of no clinical significance and may be treated with salicylates in most cases. The pathogenesis of the reaction is unclear, but it may be due to liberation of antigens from the spirochetes.

A 54-year-old female presents with a complaint of dizziness. Two days ago, while riding in a friend's car and trying to read a book, she experienced sudden extreme nausea and a "spinning" feeling that lasted for 20 minutes. She also had a headache that mainly felt like a fullness in the area around her left ear. Since then she has had only mild dizziness when she moves her head too quickly. She recalls experiencing these symptoms on two other occasions but cannot remember the circumstances, although she thinks one episode may have been related to having had too much caffeine. A review of systems is positive for a humming in her ears over the last few years. On examination both ears appear normal. Mild horizontal nystagmus can be seen on movement of the head to the left. Audiograms are normal in the right ear, with a low-frequency hearing loss on the left. Which one of the following is the most likely diagnosis? (check one) A. Motion sickness B. Meniere's disease C. Vestibular migraine D. Benign positional vertigo

Correct This patient's symptoms are compatible with Meniere's disease, which is characterized by multiple episodes of vertigo lasting for 20-120 minutes, accompanied by a fluctuating hearing loss, tinnitus, and a sense of aural fullness. Audiograms will reveal a low-frequency hearing loss with an upsloping curve, which can become flattened over the years. Most patients develop unilateral symptoms, and many patients will develop bilateral disease many years after the onset of the unilateral symptoms. Multiple studies have reported the rate of bilateral Meniere's disease to be as high as 50% many years after the initial diagnosis. Motion sickness is a common cause of nausea, but the nausea usually does not come on suddenly and is not as pronounced as with Meniere's disease. A vestibular migraine can present like a sudden Meniere's disease attack but in this patient the audiograms, tinnitus, and aural fullness suggest Meniere's disease. Benign positional vertigo is very common, and hearing loss could be an incidental finding. However, the most common form of age-related hearing loss is seen at the higher frequencies. Positional vertigo like this patient has is common between attacks of Meniere's disease. There is often a family history of Meniere's disease, and there is frequently an association with allergies. The condition can also get worse with caffeine use. Even though the diagnosis is clinical, MRI and blood tests are recommended to rule out other conditions that may be putting pressure on the endolymphatic system and thus causing the symptoms.

A morbidly obese 68-year-old male complains of breast enlargement. He has not noticed any pain or discomfort from this problem. His past medical history is negative except for type 2 diabetes mellitus and hypertension. His medications include metformin (Glucophage), 1000 mg twice daily; lisinopril (Prinivil, Zestril), 20 mg daily; and aspirin, 81 mg daily. His family history is negative for breast cancer. A physical examination is negative except for a BMI of 45 kg/m2 and symmetric bilateral adipose tissue in the breast region on inspection and palpation. There is no glandular tissue on careful palpation of the area beneath the areolae and nipples. No nodules or axillary nodes are detected. There is no nipple retraction or discharge, and no skin changes. Which one of the following is the most likely cause of this problem? (check one) A. Fat necrosis B. Gynecomastia C. Pseudogynecomastia D. Breast cancer E. Mastitis

Correct This patient most likely has pseudogynecomastia due to increases in subareolar fat secondary to his obesity. This is based upon clinical findings of symmetric adipose tissue in the breast region bilaterally and a lack of firm, palpable glandular tissue in the nipple and areolar region. In gynecomastia, there is palpable, firm glandular tissue in a concentric mass around the nipple-areola complex. Hard, immobile masses, masses associated with skin changes, nipple retraction, nipple discharge, or enlarged lymph nodes would suggest possible malignancy. Fat necrosis would involve a history of breast region trauma and would generally be asymmetric. Mastitis would cause clinical signs of infection

Staff members in your practice often complain about one of your patients. He exhibits odd behaviors and beliefs, and is always very anxious about his visit and about when he will be seen, despite long familiarity with your practice. Which one of the following personality disorders best fits the description of this patient? (check one) A. Antisocial B. Borderline C. Dependent D. Narcissistic E. Schizotypal

Correct This patient most likely has schizotypal personality disorder. These patients have problems with social and interpersonal relationships, which are marked by significant anxiety and discomfort, and they also exhibit odd thinking, speech, and perceptions. This disorder is classified as being in the cluster A personality disorder group. Patients with disorders in this group exhibit odd or eccentric personalities, and the group includes paranoid, schizoid, and schizotypal personality disorders. Cluster B disorders are characterized by dramatic, emotional, or erratic personalities, and include antisocial, borderline, histrionic, and narcissistic personality disorders. Cluster C disorders include avoidant, obsessive-compulsive, and dependent personality disorders. Patients with disorders in this group exhibit mainly anxious or fearful behaviors.

A 39-year-old male presents to the emergency department with a 2-hour history of chest discomfort, dyspnea, dizziness, and palpitations. He has no history of coronary artery disease. He states that he has had several similar episodes in the last year. On examination he has a temperature of 36.8°C (98.2°F), a respiratory rate of 25/min, a heart rate of 193 beats/min, a blood pressure of 134/82 mm Hg, and an O2 saturation of 96% on room air. The physical examination is otherwise normal. An EKG reveals a regular narrow QRS complex tachycardia with no visible P waves. He converts to normal sinus rhythm with intravenous adenosine (Adenocard). Which one of the following would be most useful in the long-term management of this patient's condition? (check one) A. Adenosine B. Digoxin C. Vagal maneuvers D. Pacemaker placement E. Radiofrequency ablation

Correct This patient presents with a classic description of supraventricular tachycardia (SVT). The initial management of SVT centers around stopping the aberrant rhythm. In the hemodynamically stable patient initial measures should include vagal maneuvers (SOR C), intravenous adenosine or verapamil (SOR B), intravenous diltiazem or β-blockade, intravenous antiarrhythmics, or cardioversion in refractory cases. While digoxin is occasionally useful in atrial fibrillation with a rapid ventricular rate, it is not recommended for SVT. Radiofrequency ablation is fast becoming the first-line therapy for all patients with recurrent SVT, not just those refractory to suppressive drug therapies. Observational studies have shown that this therapy results in improved quality of life and lower cost as compared to drug therapy (SOR B).

72-year-old female who remains very active and engaged in the community comes to your office concerned by urinary symptoms that disrupt her life. She reports that she often has a strong, abrupt desire to void that frequently causes her to leak urine involuntarily. She also reports occasional episodes of urinary frequency and nocturia. Which one of the following is the first-line treatment for her condition? (check one) A. Anticholinergic drugs such as oxybutynin or solifenacin (Vesicare) B. β-Adrenergic agonists such as mirabegron (Myrbetriq) C. Duloxetine (Cymbalta) D. Bladder training E. A pessary

Correct This patient suffers from urge urinary incontinence, defined as the loss of urine accompanied or preceded by a strong impulse to void. It may be accompanied by frequency and nocturia, and is common in older adults. Conservative therapies such as behavioral therapy, including bladder training and lifestyle modification, should be the first-line treatment for both stress and urge urinary incontinence (SOR C).Pharmacologic interventions should be used as an adjunct to behavioral therapies for refractory urge incontinence (SOR C). Vaginal inserts, such as pessaries, can be used for treating stress incontinence but not urge incontinence.

A 75-year-old female with a 10-year history of type 2 diabetes mellitus presents with moderate bilateral burning pain in the distal portion of her feet. Her hemoglobin A1c is 8.1%. Which one of the following is recommended as first-line therapy for improvement of this patient's pain? (check one) A. Amitriptyline B. Ibuprofen C. Pregabalin (Lyrica) D. Tramadol (Ultram)

Correct This patient's condition is consistent with distal symmetric polyneuropathy (DSPN). It may be present in up to 10%-15% of newly diagnosed patients with type 2 diabetes mellitus and in up to 50% of patients within 10 years of diagnosis. Pregabalin or duloxetine is recommended as the initial approach in the symptomatic treatment of neuropathic pain in diabetes (SOR A). There is no significant evidence supporting glycemic control or lifestyle interventions as effective treatment for the condition. Narcotics, including tramadol, are not first- or second-line choices, and although tricyclic antidepressants such as amitriptyline are effective, they present a higher risk for serious side effects, especially in the elderly. There are no recommendations for the use of NSAIDs.

Which one of the following is an appropriate treatment for tinea capitis? (check one) A. Oral cephalosporins B. Oral griseofulvin C. Topical acyclovir (Zovirax) D. Topical ketoconazole (Nizoral) E. Topical miconazole (Monistat)

Correct Tinea capitis is an infection of the scalp caused by a variety of superficial dermatophytes. The treatment of choice for this infection is oral griseofulvin. It has the fewest drug interactions, a good safety record, and anti-inflammatory properties. Terbinafine has equal effectiveness and requires a significantly shorter duration of therapy, but it is only available in tablet form. Since tinea capitis most commonly occurs in children, tablets would have to be cut and/or crushed prior to administration. Oral itraconazole, fluconazole, and ketoconazole have significant side effects. Topical antifungals such as ketoconazole and miconazole are ineffective against tinea capitis. Topical acyclovir is used in the treatment of herpesvirus infections, and oral cephalosporins are used in the treatment of bacterial skin infections.

A 58-year-old postmenopausal female presents with a recent onset of painless vaginal bleeding. Her last menses occurred 8 years ago and she has had no bleeding until now. She reports that her Papanicolaou smears have always been normal, with the last one obtained a year ago. A pelvic examination today is normal. Which one of the following management options is the preferred next diagnostic step? (check one) A. Colposcopy with endocervical curettage B. Transvaginal ultrasonography C. Saline infusion sonohysterography D. Hysteroscopy

Correct Transvaginal ultrasonography is the preferred initial test for a patient with painless postmenopausal bleeding, although endometrial biopsy is an option if transvaginal ultrasonography is not available. Transvaginal ultrasonography showing an endometrial thickness <3-4 mm would essentially rule out endometrial carcinoma (SOR C). An endometrial biopsy is invasive and has low sensitivity for focal lesions. Saline infusion hysterography should be considered if the endometrial thickness is greater than the threshold, or if an adequate measurement cannot be obtained by ultrasonography. If hysterography shows a global process, then a histologic diagnosis can usually be obtained with an endometrial biopsy, but if a focal lesion is present hysteroscopy should be considered as the next diagnostic step. Colposcopy is not indicated given the patient's normal Papanicolaou smear.

Which one of the following basal cell carcinomas is associated with the highest risk of recurrence? (check one) A. A 7-mm lesion on the nose B. A 9-mm lesion on the forehead C. A 12-mm lesion on the shoulder D. A 17-mm lesion on the arm

Correct Treating basal cell carcinoma with Mohs micrographic surgery leads to the lowest recurrence rate. Because of its cost and limited availability, however, this procedure should be limited to tumors with a higher risk for recurrence. Risk factors include larger size, more invasive histologic subtypes (micronodular, infiltrative, and morpheaform), and sites associated with a higher risk of recurrence. High-risk locations include the "mask" areas of the face, which include the central face, eyelids, eyebrows, periorbital area, nose, lips (cutaneous and vermilion), chin, mandible, preauricular and postauricular skin/sulci, temple, and ear. Other high-risk sites include the genitalia, hands, and feet. Moderate-risk locations include the cheeks, forehead, scalp, and neck. All other areas, including the trunk and extremities, are low-risk areas. Even with a low-risk location, a lesion that is ≥20 mm in size has a high risk of recurrence. With a moderate-risk location a lesion ≥10 mm in size carries a higher risk of recurrence, and a lesion ≥6 mm in size is considered high risk in a high-risk location.

A 6-month-old male is brought in for a routine checkup. Only one testicle is palpable. The genital examination is otherwise within normal limits. Which one of the following would be most appropriate at this time? (check one) A. Observation only, until 18 months of age B. Abdominal ultrasonography C. Urologic referral for surgical exploration D. HCG treatment for 3 months

Correct Treatment for a unilateral undescended testis should be started at 6-12 months of age to avoid testicular damage. It was once thought that delaying descent lowered the incidence of testicular cancer, but it is now believed that orchiopexy allows for early cancer detection. HCG treatment may promote descent into the distal canal, but the testicle often ascends again. Ultrasonography will not show an undescended testis in many cases and is therefore not recommended. Hormonal treatments have been used in Europe but randomized, controlled trials have not shown them to be effective.

A heroin overdose is most likely to cause acute? (check one) A. Renal failure B. Hepatic necrosis C. Myocardial infarction D. Pulmonary edema E. Pelvic thrombophlebitis

Correct Until recently, the number of heroin overdoses had been in decline for the past few decades. Although heroin still only accounts for about 1% of drug overdoses, it has become more common in the past few years. Overdose is manifested by CNS depression and hypoventilation. Clinical clues include pupillary miosis and a decreasing respiratory rate in the presence of a semi-wakeful state. In addition to hypoventilation, a multifactorial acute lung injury occurs within 2-4 hours of the overdose and is associated with hypoxemia and a hypersensitivity reaction, resulting in noncardiogenic pulmonary edema. Findings include hypoxia, crackles on lung auscultation, and pink, frothy sputum. Treatment must include respiratory support with intubation, mechanical ventilation, and oxygen, as well as opiate reversal with naloxone, which may require repeat doses or intravenous infusion. Arrhythmias and myocardial ischemia/infarction do not occur as direct pharmacologic effects of heroin, although they may occur as a consequence of the pulmonary toxicity or the presence of other drugs taken intentionally or otherwise (i.e., heroin cut with other agents). Acute renal injury, hepatic injury, and thromboembolic events are also not a direct result of the pharmacologic effects of heroin.

A 69-year-old female presents with her first episode of Clostridium difficile colitis, which is characterized as severe. Which one of the following is the most appropriate initial therapy? (check one) A. Oral metronidazole (Flagyl) B. Intravenous metronidazole C. Oral vancomycin (Vancocin) D. Intravenous vancomycin E. Rifaximin (Xifaxan)

Correct Vancomycin, 125 mg orally 4 times daily for 10-14 days, is recommended for the first severe episode of Clostridium difficile colitis (SOR B). If the first episode is mild to moderate, oral metronidazole, 500 mg3 times daily for 10-14 days, would be preferred. Intravenous vancomycin is not effective in the treatment of colitis. Rifaximin is not well studied and is not recommended in any current guidelines.

A 29-year-old female presents with redness of her left eye. She has just returned from a summer beach vacation with her children and woke up with a red eye. Your examination reveals a watery discharge, a hyperemic conjunctiva, and a palpable preauricular lymph node. Her cornea is clear on fluorescein staining. Which one of the following is most appropriate for this patient? (check one) A. Reassurance only B. Culture-guided antibiotic therapy C. Quinolone eyedrops D. Corticosteroid/antibiotic eyedrops E. Urgent ophthalmologic referral

Correct Viruses cause 80% of infectious conjunctivitis cases and viral conjunctivitis usually requires no treatment. Bacterial conjunctivitis is associated with mattering and adherence of the eyelids. Topical antibiotics reduce the duration of bacterial conjunctivitis but have no effect on viral conjunctivitis. Allergic conjunctivitis would be more likely if the patient reported itching. Antibiotics or corticosteroids would not be helpful in this patient, and would not prevent complications. The majority of cases of viral conjunctivitis are caused by adenoviruses, which cause pharyngeal conjunctival fever and epidemic keratoconjunctivitis. Pharyngeal conjunctival fever is characterized by high fever, pharyngitis, and bilateral eye inflammation. Keratoconjunctivitis occurs in epidemics, and is associated with a watery discharge, hyperemia, and ipsilateral lymphadenopathy in >50% of cases.

A 7-year-old Hispanic female has a 3-day history of a fever of 104.0°F (40.0°C), muscle aches, vomiting, anorexia, and headache. Over the past 12 hours she has developed a painless maculopapular rash that includes her palms and soles but spares her face, lips, and mouth. She has recently returned from a week at summer camp in Texas. Her pulse rate is 140 beats/min, and her blood pressure is 70/40 mm Hg. Which one of the following is the most likely diagnosis? (check one) A. Mucocutaneous lymph node syndrome B. Leptospirosis C. Rocky Mountain spotted fever D. Scarlet fever E. Toxic shock syndrome

Correct While all of the conditions listed are in the differential diagnosis, the most likely in this patient is Rocky Mountain spotted fever (RMSF) (SOR C). It is transmitted by ticks and occurs throughout the United States, but is primarily found in the South Atlantic and South Central states. It is most common in the summer and with exposure to tall vegetation from activities such as camping, hiking, or gardening. The diagnosis is based on clinical criteria that include fever, hypotension, rash, myalgia, vomiting, and headache (sometimes severe). The rash associated with RMSF usually appears 2-4 days after the onset of fever and begins as small, pink, blanching macules on the ankles, wrists, or forearms that evolve into maculopapules. It can occur anywhere on the body, including the palms and soles, but the face is usually spared. Mucocutaneous lymph node syndrome is a similar condition in children (usually <2 years old), but symptoms include changes in the lips and oral cavity, such as strawberry tongue, redness and cracking of the lips, and erythema of the oropharyngeal mucosa. Leptospirosis is usually accompanied by severe cutaneous hyperesthesia. The patient with scarlet fever usually has prominent pharyngitis and a fine, papular, erythematous rash. Toxic shock syndrome may present in a similar fashion, but usually in postmenarchal females.

Which one of the following medications used in the treatment of osteoporosis can also be used to treat the pain associated with acute and chronic vertebral compression fractures? (check one) A. Calcitonin-salmon (Miacalcin) B. Raloxifene (Evista) C. Risedronate (Actonel) D. Teriparatide (Forteo) E. Zoledronic acid (Reclast)

Correct While all of the medications listed can be used to treat osteoporosis, only calcitonin-salmon is useful in the management of pain associated with acute or chronic vertebral fractures. Calcitonin is an antiresorptive agent that has been shown to decrease the risk of vertebral fractures, but it is not considered a first-line treatment for osteoporosis because there are more effective agents. However, it does have modest analgesic properties that make it useful in the treatment of the pain associated with vertebral fractures.

A 55-year-old female receives a gynecologic and breast examination from a nurse practitioner, who also orders a routine mammogram. Who is legally responsible for ensuring that the patient is notified of the results of the mammogram? (check one) A. The nurse practitioner B. The supervising physician C. The facility performing the mammogram D. The patient

Correct While it is certainly appropriate for the nurse practitioner or physician who ordered the test to notify the patient of mammography results, the facility performing the test is legally responsible. This is specified by the federal Mammography Quality Standards Act, first passed by Congress in 1992.

An 88-year-old male nursing-home patient is having problems with constant overflow incontinence. Intermittent catheterization has proven difficult due to urethral obstruction and his resistance to such procedures. He has dementia and generalized weakness as a result of multiple strokes and is bedbound, requiring total care for most activities of daily living. Examination shows a grade 3 coccygeal ulcer that has been present for several months, and a digital rectal examination demonstrates a large, irregular prostate. Which one of the following is the best choice to quickly correct his incontinence? (check one) A. Doxazosin (Cardura) B. Finasteride (Proscar) C. Tolterodine (Detrol) D. Long-term indwelling Foley catheter placement E. Referral for transurethral prostatectomy

Correct While it would increase the risk of urinary infection, indwelling catheter placement is most likely to provide immediate relief of this patient's urinary retention. It will minimize or prevent further contamination of his decubitus ulcer with urine. Prostatectomy may relieve the urethral obstruction, but this patient is likely to remain incontinent due to his vascular dementia. Doxazosin or finasteride would likely be inadequate in this situation. Tolterodine is not indicated for overflow incontinence.

A 60-year-old male smoker has lung cancer, and a life expectancy of 4-6 months. His wife is concerned about his state of mind and requests medication for him. His cancer-related pain is generally controlled. When evaluating the patient, which one of the following features would be more characteristic of depression as opposed to a grief reaction? (check one) A. Insomnia B. Loss of interest or pleasure in all activities C. Feelings of guilt D. Thoughts of wanting to die E. Psychomotor agitation

Correct While there is significant overlap in the symptoms of each condition, there are some signs and symptoms that help the family physician determine whether a terminally ill patient is experiencing grief or has major depression. This distinction is important because the terminally ill patient with depression would likely benefit from antidepressant medication, whereas a patient with end-of-life grief is generally best managed without psychotherapeutic medications. The key clinical feature in distinguishing the two conditions is in the pervasiveness of symptoms in depression, particularly the loss of pleasure or interest in all activities. Episodic feelings of guilt, anxiety, and helplessness, and even thoughts of wanting to die can and do occur with grief reactions, but these feelings are not constant and over time the symptoms gradually wane. Terminally ill patients with major depression feel helplessly hopeless all the time, but they often respond to and significantly benefit from antidepressant medication (SOR A).

Which one of the following is most characteristic of the pain associated with acute pericarditis? (check one) A. Improvement when sitting up and leaning forward B. Improvement when lying supine C. Worsening with the Valsalva maneuver D. Radiation to the right scapula E. Radiation to both arms

Correct While there is substantial overlap in the signs, symptoms, and physical findings for the various etiologies of chest pain, a good history and physical examination can help determine which patients require immediate further evaluation for a potentially serious cause. The chest pain associated with pericarditis is typically pleuritic, and is worse with inspiration or in positions that put traction on the pleuropericardial tissues, such as lying supine. Patients with acute pericarditis typically get relief or improvement when there is less tension on the pericardium, such as when sitting and leaning forward. This position brings the heart closer to the anterior chest wall, which incidentally is the best position for hearing the pericardial friction rub associated with acute pericarditis. Radiation of chest pain to both arms should raise concerns about myocardial ischemia or infarction. Radiation to the right scapula is sometimes seen with cholelithiasis. Worsening pain with the Valsalva maneuver is nonspecific and is of no particular diagnostic value.

A mother calls to ask your advice because her healthy 3-year-old, who has not been immunized against hepatitis A, attends day care with a child who was just diagnosed with the illness. You advise her that her child should receive? (check one) A. no prophylactic treatment B. hepatitis A vaccine only C. hepatitis A vaccine and immunoglobulin D. hepatitis A vaccine, along with other family members

Correct Workers and children at child care centers should receive postexposure prophylaxis if one or more cases of hepatitis A is found in a child or worker. Hepatitis A vaccine is preferred over immunoglobulin because of its long-lasting effect, ease of administration, and efficacy. Children younger than 1 year of age should receive immunoglobulin. Family members should receive prophylaxis only during an outbreak and if their child is still in diapers.

A 76-year-old male with metastatic cancer, diabetes mellitus, and stage IV chronic renal disease develops confusion and myoclonus. His current medications include enalapril (Vasotec), 10 mg/day; glipizide (Glucotrol), 10 mg/day; and morphine sulfate, 30 mg every 4 hours for pain. The morphine was started 4 weeks ago and the dosage was gradually increased until the pain was controlled. Which one of the following is the most likely cause of his symptoms? (check one) A. A drug-drug interaction B. Metastasis to the lumbar spine C. Diabetic neuropathy D. Toxic metabolites of morphine

Correct - Palliative Morphine should be avoided in patients with renal insufficiency because the toxic metabolites morphine-3-glucuronide and morphine-6-glucuronide are not eliminated by the kidneys. Accumulation of these metabolites causes neuroexcitatory effects, including confusion, sedation, respiratory depression, and myoclonus.

An elderly male who has an implanted cardioverter-defibrillator is admitted to long-term care. He has several chronic comorbidities, including hypertension, a previous stroke, coronary artery disease, osteoarthritis, advanced chronic systolic heart failure, chronic kidney disease with a calculated glomerular filtration rate of 20 mL/min/1.73 m2, diabetes mellitus, and hypercholesterolemia. The patient's quality of life has declined to the point that he wishes to receive only palliative care. He does not want aggressive treatments, including hospitalization, except for reasons of comfort. He has decided he does not wish to be resuscitated, including CPR or intubation. When considering his goals, and after consultation with the patient and his spouse, which one of the following would be most appropriate for managing his defibrillator? (check one) A. Adjust the defibrillator to deliver shocks only for ventricular fibrillation B. Adjust the defibrillator to deliver shocks only for a heart rate >140 beats/min C. Remove the defibrillator generator D. Deactivate the defibrillator E. Make no change to the defibrillator

Correct - can also be used for Hospice and Palliative Care It is recommended that an implanted cardioverter-defibrillator be deactivated when it is inconsistent with the care goals of the patient and family. In about one-quarter of patients with an implanted cardioverter-defibrillator, the defibrillator delivers shocks in the weeks preceding death. For patients with advanced irreversible disease, defibrillator shocks rarely prevent death, may be painful, and are distressing to caregivers and family members. Advance care planning discussions should include the option of deactivating the implanted cardioverter-defibrillator when it no longer supports the patient's goals.

Which one of the following is one of the five basic principles of the patient-centered medical home? (check one) A. Utilizing the latest research and advances in treatment and diagnosis B. Coordinating a patient's care across all elements of the health care system C. Acting as a gatekeeper to limit access to specialist care D. Serving as the base of a pyramid in support of a complex health care system E. Transitioning away from delivering care in an office, and focusing on meeting patients in their own

Correct - can also be used for Hospice and Palliative Care The concept of a medical home was first suggested by the American Academy of Pediatrics in 1967 to describe the ideal care of children with disabilities. In 2004 the Future of Family Medicine Project adapted this concept to describe how primary care should be based on "continuous, relationship-centered, whole-system, comprehensive care for communities." In 2007 all of the major primary care organizations collaborated to define the foundational principles of the patient-centered medical home (PCMH). These principles include the following: Comprehensiveness: Most preventive, acute, and chronic care for individual patients can be performed at the PCMH. Patient Centered: The PCMH provides care that is relationship-based, with an orientation toward the whole person. Coordination: The PCMH coordinates care for patients across all elements of the health care system. Accessibility: The PCMH works to provide patients with timely access to providers. Quality: The PCMH continuously works to improve care quality and safety.

Which one of the following cardiovascular changes is a recognized age-related effect? (check one) A. Decreased maximal heart rate with exercise B. Decreased myocardial collagen C. Decreased myocardial mass D. Increased left ventricular compliance E. Increased heart rate at rest

Maximal heart rate with exercise generally decreases with age. A frequently used formula for predicting maximal heart rate is 220 minus age, with a correction factor of 0.85 often applied for females, who have a lower peak heart rate and a more gradual decline. Myocardial collagen and mass both increase with age. The increase in collagen may play a role in decreasing left ventricular compliance. The resting heart rate, like the maximal exercising heart rate, decreases with normal aging. Tachycardia at rest may suggest a pathologic state.

A 52-year-old female with morbid obesity is incidentally noted to have mildly elevated AST (SGOT) levels. She does not consume alcohol and denies using recreational drugs. A workup for chronic viral hepatitis and hemochromatosis is negative.Which one of the following is most likely to improve her hepatic condition? (check one) A. Pentoxifylline B. Simvastatin (Zocor) C. L-carnitine D. Vitamin E E. Weight loss

Nonalcoholic fatty liver disease is characterized by the accumulation of fat in hepatocytes. It is associated with insulin resistance, central adiposity, increased BMI, hypertension, and dyslipidemia. An incidentally discovered elevated AST level in the absence of alcohol or drug-induced liver disease strongly suggests the presence of nonalcoholic fatty liver disease. The goal of therapy is to prevent or reverse hepatic injury and fibrosis. Diabetes mellitus, hypertension, dyslipidemia, and other comorbid conditions should be appropriately managed. A healthy diet, weight loss, and exercise are first-line therapeutic measures to reduce insulin resistance in patients with nonalcoholic fatty liver disease. Weight loss has been shown to both normalize AST levels and improve hepatic histology. Vitamin E has been shown to improve AST levels but has no impact on liver histology, and pentoxifylline, simvastatin, and L-carnitine have not been shown to consistently improve either AST levels or liver histology (SOR B).

A 32-year-old white primigravida has a stillbirth at 33 weeks gestation. Which one of the following is the most likely cause? (check one) A. Infection B. Placental disease C. A fetal structural disorder D. A hypertensive disorder

Stillbirth is defined as fetal death occurring at or after 20 weeks gestation, and affects approximately 1 in 160 pregnancies in the United States. A large study of stillbirths from 2006 to 2008 tried to establish a cause in 663 cases, and a probable or possible cause was identified in approximately 75% of these. While there were some significant ethnic differences, placental abnormalities and obstetric complications were the largest category of causes in white women, and this was even more true after 32 weeks gestation. Other important causes included infection and fetal defects. More than one cause was found in one-third of cases. Ref: Stillbirth Collaborative Research Network Writing Group: Causes of death among stillbirths. JAMA 2011;306(22):2459-2468.

A 59-year-old male college professor presents with a 2-month history of right medial knee pain. There is no history of injury or overuse. He has no other specific joint pain except for occasional myalgias and arthralgias of his legs and arms. On most days he has morning stiffness lasting 15-20 minutes after getting out of bed. A review of systems is otherwise negative. On examination the right knee has full range of motion. There is tenderness at the medial joint line, but no clicking or ligamentous instability. There is crepitus with movement in both knees. Which one of the following diagnostic tests would be most appropriate at this time? (check one) A. Serologic testing B. Synovial fluid analysis C. Plain radiography D. MRI without contrast E. MRI with contrast

The most likely diagnosis for this patient's knee pain is osteoarthritis. While he is likely to have disease in both knees, it is common for patients to have unilateral symptoms, especially early on. Although osteoarthritis is mainly a clinical diagnosis, plain radiography is the diagnostic study of choice if there is concern about other diagnostic possibilities. Narrowing of the medial compartment of the knee joint is typically the first radiographic finding; osteophytes are also commonly seen on plain films. In the scenario presented here, there is no need for laboratory testing at this time.

Correct In 50%-90% of patients, pityriasis rosea starts with an erythematous, scaly, oval patch a few centimeters in diameter. This is usually followed within a few days by smaller patches on the trunk and sometimes the proximal extremities. Pityriasis rubra pilaris is a rare disease with five types. The classic adult type begins with a small red plaque on the face or upper body that gradually spreads to become a generalized eruption. The other conditions listed typically begin with multiple lesions.

Which one of the following is associated with a herald patch? (check one) A. Pityriasis alba B. Pityriasis lichenoides C. Pityriasis rosea D. Pityriasis rubra pilaris E. Pityriasis (tinea) versicolor


Conjuntos de estudio relacionados

Essential Words for Ielts Unit 4: Culture

View Set

Micro Lab 7 Bacteriophage, Fermentation, Environmentall Effects I and Antimicrobials

View Set

Chapter 2: Beginnings of English America, 1607-1660

View Set

automotive electrical I Chapter 5

View Set

Week 5 - Decision Making Under Uncertainty

View Set

Unmanned Aircraft Safety Guidelines

View Set